Download as pdf or txt
Download as pdf or txt
You are on page 1of 60

14

Electrostatics
SECTION-1
True/False
1. The work done in carrying a point charge from one point to Assertion and Reason
another in an electrostatic field depends on the path along Mark your answer as
which the point charge is carried. (1981) (a) If Assertion is true, Reason is true; Reason is the correct
2. A small metal ball is suspended in a uniform electric field explanation for Assertion.
with the help of an insulated thread. If high energy X-ray (b) If Assertion is true, Reason is true; Reason is not a
beam falls on the ball, the ball will be deflected in the correct explanation for Assertion.
direction of the field. (1983) (c) If Assertion is true; Reason is false.
(d) If Assertion is false; Reason is true.
3. Two identical metallic spheres of exactly equal masses are
taken. One is given a positive charge Q coulomb and the 7. Assertion For practical purposes, the earth is used as a
other an equal negative charge. Their masses after charging reference at zero potential in electrical circuits.
are different. (1983) Reason The electrical potential of a sphere of radius R with
4. Two protons A and B are placed in between the two plates of a charge Q uniformly distributed on the surface is given by
parallel plate capacitor charged to a potential difference V as Q
. (2008)
shown in the figure. The forces on the two protons are 4 πε 0 R
identical. (1986)
+
+
Fill in the Blanks
+ 8. Figure shows lines of constant potential in
+
+ a region in which an electric field is
A
+ present. The values of the potential are
+ written in brackets. Of the points A, B and B
+ C, the magnitude of the electric field is (50V)
+ B (40V)
greatest at the point…… . (1984) C
+A (30V)
+ (20V)
+ (10V)
+ 9. Five identical capacitor plates, each of area A, are arranged
+
such that adjacent plates are at a distance d apart, the plates
+
V are connected to a source of emf V as shown in the figure.
(1984)
5. A ring of radius R carries a uniformly distributed charge + Q. –
A point charge –q is placed on the axis of the ring at a +
distance 2R from the centre of the ring and released from rest.
The particle executes a simple harmonic motion along the
axis of the ring. (1988)
1 2 3 4 5

6. An electric line of force in the x-y plane is given by the


equation x2 + y2 = 1 . A particle with unit positive charge,
initially at rest at the point x = 1, y = 0 in the x-y plane, will
move along the circular line of force. (1988) The charge on plate 1 is …… and on plate 4 is …… .
380 Electrostatics

10. Two small balls having equal positive charges Q (coulomb) 17. Two equal negative charges − q are fixed at points (0,– a) and
on each are suspended by two insulating strings of equal (0, a) on y-axis. A positive charge Q is released from rest at
length L (metre) from a hook fixed to a stand. The whole the point (2a, 0) on the x-axis. The charge Q will (1984)
setup is taken in a satellite into space where there is no (a) execute simple harmonic motion about the origin
gravity (state of weightlessness). The angle between the (b) move to the origin and remain at rest
strings is …… and the tension in each string is ……newton. (c) move to infinity
(1986) (d) execute oscillatory but not simple harmonic motion
11. Two parallel plate capacitors of capacitances C and 2C are 18. A charge q is placed at the centre of the line joining two equal
connected in parallel and charged to a potential difference V. charges Q. The system of the three charges will be in
The battery is then disconnected and the region between the equilibrium if q is equal to (1987)
plates of capacitor C is completely filled with a material of Q Q Q Q
dielectric constant K. The potential difference across the (a) − (b) − (c) + (d) +
2 4 4 2
capacitors now becomes…. (1988)
19. A solid conducting sphere having a charge Q is surrounded by
12. A point charge q moves from point P to point S along the path
an uncharged concentric conducting hollow spherical shell.
PQRS (Fig.) in a uniform electric field E pointing parallel to
Let the potential difference between the surface of the solid
the positive direction of the X-axis. The coordinates of points
sphere and that of the outer surface of the hollow shell be V. If
P, Q, R and S are (a, b, 0), (2a, 0, 0) ( a , − b , 0 ) ( 0, 0, 0 )
the shell is now given a change of –3Q, the new
respectively. The work done by the field in the above process
potential difference between the same two surfaces is
is given by the expression …… (1989)
(1989)
Y (a) V (b) 2V
(c) 4V (d) – 2V
O 20. Seven capacitors each of capacitance 2µF are connected in a
S Q x 10
configuration to obtain an effective capacitance µF. Which
R E 11
of the following combination will achieve the desired result ?
(1990)
13. Five point charges, each of value +q coulomb, are placed on
five vertices of a regular hexagon of side L metre. The (a) (b)
magnitude of the force on the point charge of value – q
coulomb placed at the centre of the hexagon
is ………newton. (1992)
q q
(c) (d)

–q
q
21. Two identical thin rings, each of radius R, are coaxially
placed a distance R apart. If Q1 and Q2 are respectively the
q q charges uniformly spread on the two rings, the work done in
14. The electric potential V at any point x, y, z (all in metre) in moving a charge q from the centre of one ring to that of the
space is given by V = 4 x2 volt. The electric field at the point other is (1992)
(a) zero
(1m, 0, 2 m) is ………V/m. (1992)
q (Q1 − Q2 ) ( 2 − 1)
(b)
Objective Questions I (Only one correct option) 2 ( 4 πε 0 R )

15. An alpha particle of energy 5 MeV is scattered through 180° q 2 (Q1 + Q2 )


(c)
by a fixed uranium nucleus. The distance of closest approach ( 4 πε 0 R )
is of the order of (1981) (d) q (Q1 / Q2 ) ( 2 + 1) 2 ( 4 πε 0 R )
−10 −12 −15
(a) 1 Å (b) 10 cm (c) 10 cm (d) 10 cm 22. Two point charges +q and –q are held fixed at ( −d , 0 ) and
16. A hollow metal sphere of radius 5 cm is charged such that the ( d , 0 ) respectively of a x-y co-ordinate system. Then (1995)
potential on its surface is 10 V. The potential at the centre of
(a) the electric field E at all points on the x-axis has the same
the sphere is (1983)
direction.
(a) zero
(b) work has to be done in bringing a test charge from ∞ to the
(b) 10 V origin.
(c) same as at a point 5 cm away from the surface (c) electric field at all point on y-axis is along x-axis.
(d) same as at a point 25 cm away from the surface (d) the dipole moment is 2qd along the x-axis.
Electrostatics 381

23. A parallel plate capacitor of capacitance C is connected to a point due to a charge Q at a distance r from it to be Q / 4 πε 0 r.
battery and is charged to a potential difference V. Another Then the potential at the origin due to the above system of
capacitor of capacitance 2C is similarly charged to a potential charges is (1998)
difference 2V. The charging battery is now disconnected and q
(a) zero (b)
the capacitors are connected in parallel to each other in such a 8π ε 0 x0 ln 2
way that the positive terminal of one is connected to the q ln ( 2 )
(c) infinite (d)
negative terminal of the other. The final energy of the 4 π ε 0 x0
configuration is (1995) 30. Two identical metal plates are given positive charges Q1 and
3 2 25 2 9 2
(a) zero (b) CV (c) CV (d) CV Q2 (< Q1 ) respectively. If they are now brought close together
2 6 2 to form a parallel plate capacitor with capacitance C, the
24. The magnitude of electric field E in the annular region of a potential difference between them is (1999)
charged cylindrical capacitor (1996) (a) (Q1 + Q2 ) / 2C (b) (Q1 + Q2 ) / C
(a) is same throughout. (c) (Q1 − Q2 ) / C (d) (Q1 − Q2 ) / 2C
(b) is higher near the outer cylinder than near the inner 31. For the circuit shown, which of the following statements is
cylinder. true ? (1999)
(c) varies as 1/r where r is the distance from the axis. V1 = 30 V V2 = 20 V
(d) varies as 1/ r2 where r is the distance from the axis. + – + –
S1 C1= 2pF S3 C2= 3pF S2
25. A metallic solid sphere is placed in a uniform electric field.
The lines of force follow the path(s) shown in figure as
(1996)
(a) With S 1 closed,V1 = 15 V, V2 = 20 V
1 1 (b) With S 3 closed,V1 = V2 = 25 V
2 2 (c) With S 1 and S 2 closed,V1 = V2 = 0
(d) With S 3 closed,V1 = 30 V, V2 = 20 V
3 3
32. Three charges Q, +q and +q are placed at the vertices of a
4 4
right angle triangle (isosceles triangle) as shown. The net
(a) 1 (b) 2 (c) 3 (d) 4 electrostatic energy of the configuration is zero, if Q is equal
to (2000)
26. An electron of mass me , initially at rest, moves through a Q
certain distance in a uniform electric field in time t1 . A proton
of mass mp , also, initially at rest, takes time t 2 to move
through an equal distance in this uniform electric field.
Neglecting the effect of gravity, the ratio t 2 / t1 is nearly
equal to (1997) +q +q
a
(a) 1 (b) ( mp / me )1 / 2
−q − 2q
(c) ( me / mp )1 / 2 (d) 1836 (a) (b) (c) − 2q (d) +q
1+ 2 2+ 2
27. A non-conducting ring of radius 0.5 m carries a total charge 33. A parallel plate capacitor of area A, plate separation d and
of 1.11 × 10−10 C distributed non-uniformly on its capacitance C is filled with three different dielectric materials
circumference producing an electric field E everywhere in having dielectric constants K 1 , K 2 and K 3 as shown. If a
l =0
space. The value of the integral ∫ l = ∞ − E⋅ dl (l = 0 being single dielectric material is to be used to have the same
capacitance C in this capacitor then its dielectric constant K is
centre of the ring) in volt is (1997) given by (2000)
(a) +2 (b) − 1 (c) − 2 (d) zero A/2 A/2
28. A parallel combination of 0.1 MΩ resistor and a 10 µF K1 K2
d
2
capacitor is connected across a 1.5 V source of negligible d
K3
resistance. The time required for the capacitor to get charged
upto 0.75 V is approximately (in second) (1997)
(a) infinite (b) log e 2 (c) log 10 2 (d) zero A = Area of plates
29. A +q 1 1 1 1 1 1 1
charge is fixed at each of the points (a) = + + (b) = +
x = x0 , x = 3x0 , x = 5x0 K ∞ on the x-axis and a charge – q is K K 1 K 2 2K 3 K K 1 + K 2 2K 3
fixed at each of the points x = 2x0 , x = 4 x0 , x = 6x0 K , ∞. 1 K1K 2 K1K 3 K 2K 3
(c) = + 2K 3 (d) K = +
Here, x0 is a positive constant. Take the electric potential at a K K1 + K 2 K1 + K 3 K 2 + K 3
382 Electrostatics

34. A uniform electric field pointing in positive x-direction exists 39. Two equal point charges are fixed at x = − a and x = + a on
in a region. Let A be the origin, B be the point on the x-axis at the x-axis. Another point charge Q is placed at the origin. The
x = + 1cm and C be the point on the y-axis at y = + 1cm. Then change in the electrical potential energy of Q, when it is
the potentials at the points A, B and C satisfy (2001) displaced by a small distance x along the x-axis, is
(a)V A < VB (b) V A > VB approximately proportional to (2002)
(c)V A < VC (d) V A > VC (a) x (b) x2 (c) x3 (d) 1/ x
35. Three positive charges of equal value q are placed at the
40. A metallic shell has a point charge q kept inside its cavity.
vertices of an equilateral triangle. The resulting lines of force
Which one of the following diagrams correctly represents the
should be sketched as in (2001)
electric lines of forces ? (2003)

O
O
(a) (b) (a) (b)
O O O O

O (c) (d)
(c) O O (d)

41. Six charges, three positive and three P Q


36. Consider the situation shown in the figure. The capacitor A negative of equal magnitude are to be
placed at the vertices of a regular U O
has a charge q on it whereas B is uncharged. The charge R
appearing on the capacitor B a long time after the switch is hexagon such that the electric field at O
closed is (2001) is double the electric field when only
T S
q one positive charge of same magnitude
+ – is placed at R. Which of the following arrangements of charge
+ – is possible for, P, Q, R, S, T and U respectively? (2004)
+ – (a) + , − , + , − , − , + (b) + , − , + , − , + , −
+ – S (c) + , + , − , + , − , − (d) − , + , + , − , + , −
+ – 42. Assume that an electric field E = 30x2 $i exists in space. Then,
+ – the potential difference V A − VO , where VO is the potential at
A B the origin and V A the potential at x = 2 m is (2014 Main)

(a) zero (b) q/ 2 (c) q (d) 2q (a) 120 V (b) −120 V (c) −80 V (d) 80 V

37. In the given circuit, with steady current, the potential 43. In the given circuit, charge Q2 on the 2µF capacitor changes
difference across the capacitor must be (2001) as C is varied from 1µF to 3 µF.Q2 as a function of C is given
properly by (figures are drawn schematically and are not to
R
V scale) (2015 Main)

C 1µ F
V C
2µF
2R

2V E

(a) V (b) V/2 (c) V/3 (d) 2V/3 (a) (b)


38. Two identical capacitors, have the same capacitance C. One Q2 Q2
of them is charged to potential V1 and the other to V2 . Likely
C C
charged plates are then connected. Then, the decrease in 1µ F 3µF 1µF 3µF
energy of the combined system is (2002)
1 1
(a) C (V1 − V2 )
2 2
(b) C (V1 + V2 )
2 2
4 4 (c) Q2 (d) Q2
1 1
(c) C (V1 − V2 ) 2
(d) C (V1 + V2 )2 C C
4 4 1 µF 3 µF 1µF 3µF
Electrostatics 383

44. A capacitor is charged using an external battery with a (a) positive


resistance x in series. The dashed line shows the variation of (b) negative
ln I with respect to time. If the resistance is changed to 2x, the (c) zero
new graph will be (2004) (d) depends on the path connecting the initial and final
positions
49. Consider a neutral conducting sphere. A positive point charge
In I S is placed outside the sphere. The net charge on the sphere is
R then (2007)

Q
P
t

(a) P (b) Q (c) R (d) S


(a) negative and distributed uniformly over the surface of the
NOTE Here I is the charging current
sphere
45. Three infinitely long charge sheets are placed as shown in (b) negative and appears only at the point on the sphere
figure. The electric field at point P is (2005)
closest to the point charge
z
(c) negative and distributed non-uniformly over the entire
σ z=3a surface of the sphere
P (d) zero
− 2σ z=0
x 50. A circuit is connected as shown in the figure with the switch S
−σ z=–a open. When the switch is closed, the total amount of charge
2σ $ 2σ $ 4σ $ 4σ $ that flows from Y to X is (2007)
(a) k (b) − k (c) k (d) − k
ε0 ε0 ε0 ε0 3 µF 6 µF
X
46. A 4µF capacitor and a resistance of 2.5 MΩ are in series with
12 V battery. Find the time after which the potential S
difference across the capacitor is 3 times the potential 3Ω 6Ω
difference across the resistor. [Given, ln (2) = 0.693] (2005) Y
(a) 13.86 s (b) 6.93 s (c) 7 s (d) 14 s
9V
47. Find the time constant for the given RC circuits in correct
order (in µs). (2006) (a) zero (b) 54 µC
(c) 27 µC (d) 81µC
V V
R1 51. A spherical portion has been removed
C2
from a solid sphere having a charge
C1 distributed uniformly in its volume as
R2 C1 R1
shown in the figure. The electric field
inside the emptied space is (2007)
(a) zero everywhere
C2 R2 (b) non-zero and uniform
(c) non-uniform
V
(d) zero only at its centre
52. A long, hollow conducting cylinder is kept co-axially inside
R1 C1 another long, hollow conducting cylinder of larger radius.
Both the cylinders are initially electrically neutral. (2007)
R1 (a) A potential difference appears between the two cylinders
when a charge density is given to the inner cylinder.
R2 C2 (b) A potential difference appears between the two cylinders
when a charge density is given to the outer cylinder.
R1 = 1 Ω , R2 = 2 Ω , C1 = 4 µF, C 2 = 2µF.
(c) No potential difference appears between the two
(a) 18, 4, 8/9 (b) 18, 8/9, 4 (c) 4, 18, 8/9 (d) 4 , 8/ 9, 18 cylinders when a uniform line charge is kept along the
48. Positive and negative point charges of equal magnitude are axis of the cylinders.
kept at ( 0, 0, a / 2 ) and ( 0, 0, − a / 2 ) , respectively. The work (d) No potential difference appears between the two
done by the electric field when another positive point charge cylinders when same charge density is given to both the
in moved from ( −a , 0, 0 ) to ( 0, a , 0 ) is (2007) cylinders.
384 Electrostatics

53. A parallel plate capacitor C with plates of unit area and |E (r)| V (r)

separation d is filled with a liquid of dielectric constant


K = 2. The level of liquid is d / 3 initially. Suppose the liquid (c)
level decreases at a constant speed v , the time constant as a
function of time t is (2008)
0 R r
|E (r)| V (r)
C
d R
d (d)
3

0 R r

57. In the given circuit, a charge of +80 µC is given to the upper


6ε 0 R (15d + 9vt )ε 0 R
(a) (b) plate of the 4 µF capacitor. Then in the steady state, the
5d + 3vt 2d 2 − 3dvt − 9v2 t 2
charge on the upper plate of the 3 µF capacitor is (2012)
6ε 0 R (15d − 9vt ) ε 0 R
(c) (d)
5d − 3vt 2d 2 + 3dvt − 9v2 t 2 + 80 µC
54. Three concentric metallic spherical shells of radii 4 µF
R , 2 R and 3 R are given charges Q1 , Q2 and Q3 , respectively.
It is found that the surface charge densities on the outer
2 µF 3 µF
surfaces of the shells are equal. Then, the ratio of the charges
given to the shells, Q1 : Q2 : Q3 , is (2009)
(a) 1 : 2 : 3 (b) 1 : 3 : 5
(c) 1 : 4 : 9 (d) 1 : 8 : 18
(a) + 32 µC (b) + 40 µC
55. A 2 µF capacitor is charged as shown in the figure. The (c) + 48 µC (d) + 80 µC
percentage of its stored energy dissipated after the switch S is 58. Two large vertical and parallel metal plates having a
turned to position 2 is (2011)
separation of 1 cm are connected to a DC voltage source of
1 2 potential difference X . A proton is released at rest midway
between the two plates. It is found to move at 45° to the
vertical just after release. Then X is nearly (2012)
−5 −7
2 µF 8 µF (a) 1 × 10 V (b) 1 × 10 V
(c) 1 × 10−9 V (d) 1 × 10−10 V

(a) 0% (b) 20% 59. Two charges, each equal to q, are kept at x = − a and x = a on
q
(c) 75% (d) 80% the x-axis. A particle of mass m and charge q0 = − is placed
2
56. Consider a thin spherical shell of radius R with its centre at at the origin. If charge q0 is given a small displacement
the origin, carrying uniform positive surface charge density. y ( y << a ) along the y-axis, the net force acting on the
The variation of the magnitude of the electric field | E ( r )| and particle is proportional to (2013 Main)
the electric potentialV ( r ) with the distance r from the centre, (a) y (b) − y
is best represented by which graph? (2012) 1 1
(c) (d) −
|E (r)| V (r) y y
60. A charge Q is uniformly distributed over a long rod AB of
(a) length L as shown in the figure. The electric potential at the
point O lying at distance L from the end A is (2013 Main)
0 R r O A B
|E (r)| V (r) L L

Q 3Q
(a) (b)
(b) 8π ε 0 L 4 π ε0 L
Q Q ln 2
(c) (d)
0 R r 4 π ε 0 L ln 2 4 π ε0 L
Electrostatics 385

61. Charges Q , 2Q and 4Q are uniformly distributed in three 65. In the given circuit, charge Q2 on the 2µF capacitor changes
R as C is varied from 1µF to 3 µF.Q2 as a function of C is given
dielectric solid spheres 1, 2 and 3 of radii , R and 2R
2 properly by (figures are drawn schematically and are not to
respectively, as shown in figure. If magnitudes of the electric scale) (2015 Main)
fields at point P at a distance R from the centre of spheres 1, 2 1 µF
and 3 are E1 , E2 and E3 respectively, then (2014 Adv.)

P P P C
R R R 2µF
Q 2Q 4Q
R/2
2R E

Sphere-1 Sphere-2 Sphere-3


(a) E1 > E2 > E3 (b) E3 > E1 > E2 Q2 Q2
(a) (b)
(c) E2 > E1 > E3 (d) E3 > E2 > E1
C C
62. Assume that an electric field E = 30x2 $i exists in space. Then, 1µ F 3µF 1µF 3µF
the potential difference V A − VO , where VO is the potential at
the origin and V A the potential at x = 2 m is (2014 Main)
(a) 120 V (b) −120 V (c)
Q2
(d)
Q2
(c) −80 V (d) 80 V C C
1 µF 3 µF 1µF 3µF
63. A long cylindrical shell carries positive surface charge σ in
the upper half and negative surface charge − σ in the lower
half. The electric field lines around the cylinder will look like 66. A combination of capacitors is set-up as shown in the figure.
figure given in (figures are schematic and not drawn to scale) The magnitude of the electric field, due to a point charge Q
(2015 Main) (having a charge equal to the sum of the charges on the 4 µF
and 9 µF capacitors), at a point distant 30 m from it, would
equal to (2016 Main)
++++ + +++
(a) –– –+
+ – (b) –– ––+
+
–– –– 3 µF
–– –– 4 µF

9 µF

++++ ++++ 2 µF
(c) –– ––+
+ (d) –– ––+
+
–– –– –– ––

8V
64. A parallel plate capacitor having plates of area S and plate
separation d, has capacitance C1 in air. When two dielectrics (a) 240 N/C (b) 360 N/C
of different relative permittivities (ε1 = 2 and ε 2 = 4) are (c) 420 N/C (d) 480 N/C
introduced between the two plates as shown in the figure, the 67. An electric dipole has a fixed dipole moment P, which makes
C angle θ with respect to X-axis. When subjected to an electric
capacitance becomes C 2 . The ratio 2 is
C1 (2015 Adv.) field E1 = Ei$, it experiences a torque T1 = τ k$ . When
d/2
subjected to another electric field E = 3E $j, it experiences
2 1
a torque T2 = − T1 . The angle θ is (2017 Main)

S/ 2 (a) 45° (b) 60°


ε2
(c) 90° (d) 30°
68. A capacitance of 2 µF is required in an electrical circuit across
S/ 2 a potential difference of 1kV. A large number of 1 µF
ε1
capacitors are available which can withstand a potential
difference of not more than 300 V. The minimum number of
d
capacitors required to achieve this is (2017 Main)
6 5 7 7 (a) 16 (b) 24
(a) (b) (c) (d)
5 3 5 3 (c) 32 (d) 2
386 Electrostatics

69. In the given circuit diagram, when the current reaches steady  1 2 − 2
Take, 4 πε = 9 × 10 N-m C 
9
state in the circuit, the charge on the capacitor of capacitance  
0 (2019 Main)
C will be (2017 Main)
E . × 10 m / s
(a) 15 2
(b) 3.0 × 104 m / s
r
(c) 1.0 m/s (d) 2.0 × 103 m / s
r1
76. A system of three charges are placed as shown in the figure
(2019 Main)
C
D
r2 +q –q Q
r1 r2
(a) CE (b) CE d
( r2 + r ) ( r + r2 )
r1 If D >> d, the potential energy of the system is best given by
(c) CE
( r1 + r )
(d) CE 1  q 2 2qQd  1  q 2 qQd 
(a) − + 2 
(b) + + 2 
4 πε 0  d D  4 πε 0  d D 
70. Three concentric metal shells A , B and C of respective radii
a , b and c ( a < b < c ) have surface charge densities + σ , − σ 1  q 2 qQd  1  q 2 qQd 
(c) − −  (d) − − 2 
and + σ, respectively. The potential of shell B is (2018 Main) 4 πε 0  d 2D 2  4 πε 0  d D 
σ  b 2 − c2  σ  a2 − b2  77. A positive point charge is released from rest at a distance r0
(a)  + a (b)  + c
ε0  c  ε0  a  from a positive line charge with uniform density. The speed
σ a − b
2 2 σ b − c
2 
2 ( v ) of the point charge, as a function of instantaneous distance
(c)  + c (d)  + a r from line charge, is proportional to (2019 Main)
ε0  b  ε0  b 

71. A parallel plate capacitor of capacitance 90 pF is connected
to a battery of emf 20 V. If a dielectric material of dielectric
constant K = 5/ 3 is inserted between the plates, the magnitude
of the induced charge will be (2018 Main) r0
(a) 0.9 µC (b) 1.2 µC (c) 0.3 µC (d) 2.4 µC
72. Four point charges − q , + q , + q and −q are placed on Y -axis
at y = −2d , y = − d , y = + d and y = +2d , respectively. The r
+
magnitude of the electric field E at a point on the X -axis at  r
(a) v ∝   (b) v ∝ e r0
x = D, with D >> d , will behave as (2019 Main)  r0 
1 1 1 1
(a) E ∝ (b) E ∝ 3 (c) E ∝ 2 (d) E ∝ 4  r  r
D D D D (c) v ∝ ln   (d) v ∝ ln  
 r0   r0 
73. Two point charges q1 ( 10 µC ) and q2 ( − 25 µC ) are placed
78. The electric field in a region is given by E = ( Ax + B )i$,
on the x-axis at x = 1m and x = 4 m, respectively. The electric
where E is in NC−1 and x is in metres. The values of constants
field (in V/m) at a point y = 3 m on Y -axis is
are A = 20SI unit and B = 10SI unit. If the potential at x = 1is
 1 
 Take, = 9 × 109 N-m2 C−2  V1 and that at x = − 5 isV2 , thenV1 − V2 is (2019 Main)
 4πε 0  (2019 Main) (a) − 48 V (b) − 520 V (c) 180 V (d) 320 V
$ $
(a) ( 63 i − 27 j ) × 102
(b) ( 81 i$ − 81$j ) × 102
79. Three charges Q, +q and +q are placed at the vertices of a
(c) ( −81 i$ + 81$j ) × 102 (d) ( −63 $i + 27$j ) × 102 right angle isosceles triangle as shown below. The net
74. Three charges +Q , q , + Q are placed respectively at distance electrostatic energy of the configuration is zero, if the value
d of Q is (2019 Main)
0, and d from the origin on the X -axis. If the net force
2 Q
experienced by +Q placed at x = 0 is zero, then value of q is
(2019 Main)
+Q +Q −Q −Q
(a) (b) (c) (d)
2 4 2 4
75. In free space, a particle A of charge 1µC is held fixed at a +q +q
point P. Another particle B of the same charge and mass −q − 2q
4 µkg is kept at a distance of 1 mm from P. If B is released, (a) −2q (b) (c) +q (d)
1+ 2 2+1
then its velocity at a distance of 9 mm from P is
Electrostatics 387

80. Four equal point charges Q each are placed in the xy-plane at shell is now given a charge of −4 Q, the new potential
(0, 2), (4, 2), (4, −2) and ( 0, − 2 ). The work required to put a difference between the same two surfaces is (2019 Main)
fifth charge Q at the origin of the coordinate system will be (a) −2V (b) 2V (c) 4V (d) V
(2019 Main)
85. The given graph shows variation (with distance r from centre)
Q2 Q2  1
(a) (b) 1 +  of (2019 Main)
4πε 0 4 πε 0  3
Q2 Q2  1
(c) (d) 1 + 
2 2πε 0 4 πε 0  5 ro
81. A charge Q is distributed over three concentric spherical
shells of radii a , b , c ( a < b < c ) such that their surface charge
densities are equal to one another. The total potential at a ro r
point at distance r from their common centre, where r < a (a) electric field of a uniformly charged spherical shell
would be (2019 Main) (b) potential of a uniformly charged spherical shell
Q ( a 2 + b 2 + c2 ) Q(a + b + c ) (c) electric field of a uniformly charged sphere
(a) (b)
4 πε 0 ( a 3 + b 3 + c3 ) 4 πε 0 ( a 2 + b 2 + c2 ) (d) potential of a uniformly charged sphere
Q Q ab + bc + ca 86. A point dipole p = − p0 x $ is kept at the origin. The potential
(c) (d) ⋅
4 πε 0 ( a + b + c ) 12 π ε 0 abc and electric field due to this dipole on the Y-axis at a distance
d are, respectively [Take, V = 0 at infinity] (2019 Main)
82. A uniformly charged ring of radius 3a and total charge q is
p p −p
placed in xy-plane centred at origin. A point charge q is (a) , (b) 0,
moving towards the ring along the Z-axis and has speed v at 4 πε 0 d 2 4 πε 0 d 3 4 πε 0 d 3
z = 4 a. The minimum value of v such that it crosses the origin p p −p
is (c) 0, (d) ,
(2019 Main)
4 πε 0 d 3
4 πε 0 d 4 πε 0 d 3
2
1/ 2 1/ 2
2  1 q2  2  4 q2  87. Determine the electric dipole moment of the system of three
(a)   (b)  
m  5 4 πε 0 a  m  15 4 πε 0 a  charges, placed on the vertices of an equilateral triangle as
1/ 2 1/ 2
shown in the figure. (2019 Main)
2  1 q2  2  2 q2 
(c)   (d)  
m  15 4 πε 0 a  m  15 4 πε 0 a 
–2q
y
83. Shown in the figure is a shell made of a conductor. It has inner
radius a and outer radius b and carries charge Q. At its centre l l
is a dipole p as shown. In this case, (2019 Main) +q
+q
l x
$j − $i $i + $j
(a) 3 ql (b) 2ql $j (c) − 3 ql $j (d) ( ql )
p 2 2
88. An electric field of 1000 V/m is applied to an electric dipole
at angle of 45º. The value of electric dipole moment is
10− 29 C-m. What is the potential energy of the electric
dipole? (2019 Main)
(a) surface charge density on the inner surface is uniform and
(a) − 9 × 10− 20 J (b) − 10 × 10− 29 J
 Q
  (c) − 20 × 10− 18 J (d) − 7 × 10− 27 J
 2
equal to
4 πa 2 89. Two electric dipoles, A , B with respective dipole moments
(b) electric field outside the shell is the same as that of a point d A = − 4 qa i$ and dB = − 2 qa $i are placed on the X -axis
charge at the centre of the shell with a separation R, as shown in the figure
(c) surface charge density on the outer surface depends on p R
(d) surface charge density on the inner surface of the shell is X
zero everywhere A B

84. A solid conducting sphere, having a charge Q, is surrounded The distance from A at which both of them produce the same
by an uncharged conducting hollow spherical shell. Let the potential is (2019 Main)
potential difference between the surface of the solid sphere 2R 2R R R
(a) (b) (c) (d)
and that of the outer surface of the hollow shell be V. If the 2+1 2−1 2+1 2−1
388 Electrostatics

90. For a uniformly charged ring of radius R, the electric field on 94. The parallel combination of two air filled parallel plate
its axis has the largest magnitude at a distance h from its capacitors of capacitance C and nC is connected to a battery
centre. Then, value of h is (2019 Main) of voltage, V. When the capacitors are fully charged, the
R battery is removed and after that a dielectric material of
(a) (b) R 2
2 dielectric constant K is placed between the two plates of the
R first capacitor. The new potential difference of the combined
(c) R (d) system is (2019 Main)
5
( n + 1)V nV
91. In the given circuit, the charge on 4 µF capacitor will be (a) (b)
(K + n ) K +n
(2019 Main)
V
(c) V (d)
1 mF K +n
4 mF
5 mF 95. A capacitor with capacitance 5 µF is charged to 5 µC. If the
plates are pulled apart to reduce the capacitance to 2 µF, how
much work is done? (2019 Main)
3 mF (a) 6.25 × 10−6 J (b) 2.16 × 10−6 J
(c) 2.55 × 10−6 J (d) 3.75 × 10−6 J
10 V
96. A parallel plate capacitor has 1µF capacitance. One of its two
(a) 5.4 µC (b) 9.6 µC plates is given + 2µC charge and the other plate + 4µC
(c) 13.4 µC (d) 24 µC charge. The potential difference developed across the
92. A simple pendulum of length L is placed between the plates of capacitor is (2019 Main)

a parallel plate capacitor having electric field E, as shown in (a) 1 V (b) 5 V (c) 2 V (d) 3 V
figure. Its bob has mass mand charge q. The time period of the 97. Voltage rating of a parallel plate capacitor is 500 V. Its
pendulum is given by (2019 Main) dielectric can withstand a maximum electric field of
106 V/m. The plate area is 10−4 m 2 . What is the dielectric
+ –
+ – constant, if the capacitance is 15 pF?
+ L

+ – (Take, ε 0 = 8.86 × 10−12 C2 / N-m 2) (2019 Main)
+ –
+ m – (a) 3.8 (b) 8.5 (c) 4.5 (d) 6.2
+ q –
+ – 98. In the circuit shown, find C if the effective capacitance of the
+ –
+ – whole circuit is to be 0.5 µF. All values in the circuit are inµF.
+ – (2019 Main)
E C 2
A
L L
(a) 2π (b) 2π 2 2 1
2
 qE  q2E 2
g + 
2 g −
2
2
 m m2 2 2
L L B
(c) 2π (d) 2π
 qE   qE  6
g +  g −  (a) µF (b) 4 µF
 m  m 5
7 7
93. Figure shows charge ( q ) versus voltage (V ) graph for series (c) µF (d) µF
and parallel combination of two given capacitors. The 10 11
capacitances are (2019 Main) 99. A parallel plate capacitor with plates of area 1 m 2 each, are at
q(µC)
A a separation of 0.1 m. If the electric field between the plates is
100 N/C, the magnitude of charge on each plate is
 − 12 C2 
Take, ε 0 = 8.85 × 10 
500 B  N − m2  (2019 Main)
80 (a) 9.85 × 10− 10 C
10 V V(volt) (b) 8.85 × 10− 10 C
(a) 60 µF and 40 µF (b) 50 µF and 30 µF (c) 7.85 × 10− 10 C
(c) 20 µF and 30 µF (d) 40 µF and 10 µF (d) 6.85 × 10− 10 C
Electrostatics 389

100. The charge on a capacitor plate in a circuit as a function of 105. An electric dipole is formed by two equal and opposite
time is shown in the figure. (2019 Main) charges q with separation d. The charges have same mass m. It
6 is kept in a uniform electric field E. If it is slightly rotated
5
from its equilibrium orientation, then its angular frequency
ω is (2019 Main)
4
q(µC) 2qE qE qE qE
3 (a) (b) 2 (c) (d)
md md md 2md
2 106. The bob of a simple pendulum has mass 2g and a charge of
0 5.0 µC. It is at rest in a uniform horizontal electric field of
2 4 6 8
t(s) intensity 2000 V/m. At equilibrium, the angle that the
pendulum makes with the vertical is (take g = 10m/ s2)
What is the value of current at t = 4 s? (2019 Main)
(a) 2 µA . µA
(b) 15 (c) Zero (d) 3 µA (a) tan −1 ( 2.0 ) (b) tan −1 ( 0.2 )
101. In the figure shown, after the switch ‘S ’ is turned from (c) tan −1 ( 5.0 ) (d) tan −1 ( 0.5 )
position ‘A’ to position ‘B ’, the energy dissipated in the
2 1
circuit in terms of capacitance ‘C ’ and total charge ‘Q ’ is 107. Two isolated conducting spheres S 1 and S 2 of radii R and
(2019 Main) 3 3
A B R, have charges 12 µC and − 3µC respectively, and are at a
large distance from each other. They are now connected by a
conducting wire. A long time after this is done, the charges on
S
S 1 and S 2 respectively, are (2020 Main)
ε
3C (a) 4.5 µC on both (b) + 4.5 µC and − 4.5µC
C (c) 3 µC and 6 µC (d) 6 µC and 3 µC
108. In the circuit shown in the figure, the total charge is 750 µC
2 2 2 2 and the voltage across capacitor C 2 is 20 V. Then, the charge
3 Q 5 Q 1 Q 3 Q
(a) ⋅ (b) . (c) ⋅ (d) ⋅ on capacitor C 2 is (2020 Main)
4 C 8 C 8 C 8 C
C2
102. In the figure shown below, the charge on the left plate of the C1=15 µF
10 µF capacitor is − 30 µC. The charge on the right plate of
the 6 µF capacitor is (2019 Main)
C3=8 µF
6 µF
+ –
10 µF 2 µF V
4 µF
(a) + 12 µC (b) + 18 µC (c) − 12 µC (d) − 18 µC (a) 450 µC (b) 590 µC
(c) 160 µC (d) 650 µC
103. A parallel plate capacitor having capacitance 12 pF is charged
by a battery to a potential difference of 10 V between its Objective Questions II (One or more correct option)
plates. The charging battery is now disconnected and a
porcelain slab of dielectric constant 6.5 is slipped between the 109. A parallel plate air capacitor is connected to a battery. The
plates. The work done by the capacitor on the slab is quantities charge, voltage, electric field and energy
(2019 Main) associated with this capacitor are given by Q0 , V0 , E0 and U 0
(a) 560 pJ (b) 508 pJ (c) 692 pJ (d) 600 pJ respectively. A dielectric slab is now introduced to fill the
2 space between the plates with the battery still in connection.
104. A parallel plate capacitor is of area 6 cm and a separation
3 mm. The gap is filled with three dielectric materials of equal The corresponding quantities now given by Q, V, E and U are
thickness (see figure) with dielectric constants K 1 = 10, related to the previous one as (1985)
K 2 = 12 and K 3 = 14. The dielectric constant of a material (a) Q > Q0 (b)V > V0
which give same capacitance when fully inserted in above (c) E > E0 (d)U > U 0
capacitor, would be (2019 Main) 110. A parallel plate capacitor is charged and the charging battery
is then disconnected. If the plates of the capacitor are moved
farther apart by means of insulating handles (1987)
K1 K2 K3 3 mm (a) the charge on the capacitor increases
(b) the voltage across the plates increases
(c) the capacitance increases
(a) 4 (b) 36 (c) 12 (d) 14 (d) the electrostatic energy stored in the capacitor increases
390 Electrostatics

111. Capacitor C1 of capacitance 1 µF and capacitor C 2 of


A
capacitance 2 µF are separately charged fully by a common
battery. The two capacitors are then separately allowed to q B
discharge through equal resistors at time t = 0. (1989)
(a) The currents in each of the two discharging circuits is zero
at t = 0
(b) The currents in the two discharging circuits at t = 0 are (a) electric field near A in the cavity = electric field near B in
equal but not zero the cavity.
(c) The currents in the two discharging circuits at t = 0 are (b) charge density at A = charge density at B
unequal (c) potential at A = potential at B
(d) Capacitor C1 , loses 50% of its initial charge sooner than (d) total electric field flux through the surface of the cavity is
C 2 loses 50% of its initial charge q/ ε 0 .
117. Under the influence of the coulomb field of a fixed charge
112. A parallel plate capacitor of plate area A and plate separation
+ Q, a charge −q is moving around it in an elliptical orbit.
d is charged to potential difference V and then the battery is
Find out the correct statement(s). (2009)
disconnected. A slab of dielecric constant K is then inserted
(a) The angular momentum of the charge − q is constant
between the plates of the capacitor so as to fill the space
(b) The linear momentum of the charge − q is constant
between the plates. If Q, E and W denote respectively, the
(c) The angular velocity of the charge − q is constant
magnitude of charge on each plate, the electric field between
(d) The linear speed of the charge − q is constant
the plates (after the slab is inserted), and work done on the
system, in question, in the process of inserting the slab, then 118. A few electric field lines for a
ε AV ε KAV system of two charges Q1 and
(a) Q = 0 (b) Q = 0 (1991) Q2 fixed at two different Q1 Q2
d d
points on the x-axis are shown
ε AV 2
(c) E = V / Kd (d) W = 0
2d
[1 − 1/ K ] in the figure. These lines
suggest that (2010)
113. A non-conducting solid sphere of radius R is uniformly (a) | Q1 | > | Q2 |
charged. The magnitude of the electric field due to the sphere (b) | Q1 | < |Q2 |
(c) at a finite distance to the left of Q1 the electric field is zero
at a distance r from its centre (1998)
(d) at a finite distance to the right of Q2 the electric field is
(a) increases as r increases for r < R zero
(b) decreases as r increases for 0 < r < ∞
(c) decreases as r increases for R < r < ∞
119. Two non-conducting solid spheres of radii R and 2R, having
uniform volume charge densitiesρ1 andρ2 respectively, touch
(d) is discontinuous at r = R
each other. The net electric field at a distance 2R from the
114. A dielectric slab of thickness d is inserted in a parallel plate centre of the smaller sphere, along the line joining the centre
capacitor whose negative plate is at x = 0 and positive plate is of the spheres, is zero. The ratio ρ1 / ρ2 can be (2013 Adv.)
at x = 3d . The slab is equidistant from the plates. The capacitor −32
is given some charge. As x goes from 0 to 3d (1998) (a) − 4 (b)
25
(a) the magnitude of the electric field remains the same. 32
(b) the direction of the electric field remains the same. (c) (d) 4
25
(c) the electric potential increases continuously.
(d) the electric potential increases at first, then decreases and 120. In the circuit shown in the figure, there are two parallel plate
again increases. capacitors each of capacitance C. The switch S 1 is pressed
first to fully charge the capacitor C1 and then released. The
115. A positively charged thin metal ring of radius R is fixed in the
switch S 2 is then pressed to charge the capacitor C 2 . After
x-y plane with its centre at the origin O. A negatively charged some time, S 2 is released and then S 3 is pressed. After some
particle P is released from rest at the point (0,0, z 0 ) where time (2013 Adv.)
z 0 > 0. Then the motion of P is (1998) S1 S2 S3
(a) periodic for all values of z 0 satisfying 0 < z 0 < ∞
(b) simple harmonic for all values of z 0 satisfying 0 < z 0 ≤ R C1 C2
(c) approximately simple harmonic provided z 0 < < R 2V0 V0
(d) such that P crosses O and continues to move along the
negative z-axis towards z = − ∞
116. An elliptical cavity is carved within a perfect conductor. A (a) the charge on the upper plate of C1 is 2 CV 0
positive charge q is placed at the centre of the cavity. The (b) the charge on the upper plate of C1 is CV 0
points A and B are on the cavity surface as shown in the (c) the charge on the upper plate of C 2 is 0
figure. Then (1999) (d) the charge on the upper plate of C 2 is −CV0
Electrostatics 391

Passage Based Questions 3µF B 1µF


F G
Passage
The nuclear charge ( Ze ) is non-uniformly distributed within a 3µF 1µF
nucleus of radius R. The charge density ρ( r ) (charge per unit
D E
volume) is dependent only on the radial distance r from the 1µF
centre of the nucleus as shown in figure. The electric field is
10 Ω
only along the radial direction.
A C
ρ (r) 20 Ω 100 V

d 127. A charge Q is distributed over two concentric hollow spheres


of radii r and R (> r) such that the surface densities are equal.
Find the potential at the common centre. (1981)

128. Calculate the steady state current in the 2 Ω resistor shown in


r the circuit (see figure). The internal resistance of the battery is
a R
121. The electric field at r = R is negligible and the capacitance of the condenser C is 0.2 µF.
(2008) (1982)
(a) independent of a
(b) directly proportional to a 2Ω
(c) directly proportional to a 2
(d) inversely proportional to a
122. For a = 0, the value of d (maximum value of ρ as shown in the 3Ω
C 4Ω
figure) is (2008)
3Ze 3Ze
(a) (b)
4 πR 3 πR 3
4 Ze Ze 2.8 Ω
(c) (d)
3πR 3 3πR 3 V=6V
123. The electric field within the nucleus is generally observed to
be linearly dependent on r. This implies (2008) 129. Eight point charges are placed at the corners of a cube of edge
R a as shown in figure. Find the work done in disassembling
(a) a = 0 (b) a = this system of charges. (2003)
2
2R
(c) a = R (d) a = +q −q
3
−q +q
Analytical & Descriptive Questions
124. A pendulum bob of mass 80 mg and carrying a charge of −q +q
−8
2 × 10 C is at rest in a horizontal uniform electric field of
+q −q
20,000 V/m. Find the tension in the thread of the pendulum
and the angle it makes with the vertical.
130. A particle of mass 10−3 kg and charge 1.0 C is initially at rest.
(Take g = 9.8 ms −2 ) (1979) At time t = 0, the particle comes under the influence of an
125. A charged particle is free to move in an electric field. Will it electric field E ( t ) = E0 sin ωt $i , where E0 = 1.0 NC−1 and
always move along an electric line of force? (1979 ω = 103 rad s −1 . Consider the effect of only the electrical
126. Find the potential difference between the points A and B and force on the particle. Then, the maximum speed in ms −1 ,
between the points B and C in the steady state. (1980) attained by the particle at subsequent times is ...... (2019
. Adv.)
SECTION-2
Objective Questions I (Only one correct option) 5. The figures below depict two situations in which two
a infinitely long static line charges of constant positive line
1. A disc of radius having y
4 charge density λ are kept parallel to each other.
a uniformly distributed In their resulting electric field, point charges q and −q are
charge 6C is placed in the kept in equilibrium between them. The point charges are
x
x- y plane with its centre confined to move in the x direction only. If they are given a
 −a  small displacement about their equilibrium positions, then the
at  , 0, 0 . A rod of
 2  correct statements is/are
length a carrying a
uniformly distributed charge 8C is placed on the x-axis from
a 5a λ λ λ λ
x = to x = . Two point charges −7C and 3C are placed at
4 4
x
 a −a   −3a 3a  +q –q
x
 , , 0 and  , , 0 , respectively. Consider a cubical
4 4   4 4 
a a a
surface formed by six surfaces x = ± , y = ± , z = ± .The
2 2 2
(2015 Adv.)
electric flux through this cubical surface is (2009)
(a) both charges execute simple harmonic motion.
−2C 2C 10 C 12C (b) both charges will continue moving in the direction of their
(a) (b) (c) (d)
ε0 ε0 ε0 ε0 displacement.
(c) charge + q executes simple harmonic motion while
2. A uniformly charged thin spherical shell of radius R carries charge − q continues moving in the direction of its
uniform surface charge density of σ per unit area. It is made of displacement.
two hemispherical shells, held together by pressing them with (d) charge − q executes simple harmonic motion while
force F (see figure). F is proportional to (2010)
charge + q continues moving in the direction of its
displacement.
F F
6. The region between two
concentric spheres of radii a and b,
respectively (see the figure), has
1 2 2 1 2 1 σ2 1 σ2 volume charge density ρ = ,
A a
(a) σ R (b) σ R (c) (d) Q
ε0 ε0 ε0 R ε0 R 2 r
where, A is a constant and r is the
3. A tiny spherical oil drop carrying a net charge q is balanced in distance from the centre. At the b
still air with a vertical uniform electric field of strength centre of the spheres is a point
81π charge Q. The value of A, such that
× 105 Vm −1 . When the field is switched off, the drop is
7 the electric field in the region between the spheres will be
observed to fall with terminal velocity 2 × 10−3 ms −1 . Given constant, is (2016 Main)
Q Q
g = 9.8 ms −2 , viscosity of the air = 1.8 × 10−5 Ns m −2 and (a) (b)
2πa 2 2π( b 2 − a 2 )
the density of oil = 900 kg m−3 , the magnitude of q is 2Q 2Q
(c) (d) 2
(a) 1.6 × 10−19 C (b) 3.2 × 10−19 C (2010) π( a 2 − b 2 ) πa
(c) 4.8 × 10−19 C (d) 8.0 × 10−19 C
7. Let a total charge 2Q be distributed in a sphere of radius R,
4. Consider an electric field z with the charge density given by ρ( r ) = kr, where r is the
E = E0 x,$ where E0 is a constant. (a,0,a) (a,a,a)
distance from the centre. Two charges A and B, of − Q each,
The flux through the shaded area are placed on diametrically opposite points, at equal distance
(as shown in the figure) due to a, from the centre. If A and B do not experience any force,
this field is (2011) y
(0,0,0) (0,a,0) then (2019 Main)
−1 / 4 3R
x (a) a = 8 R (b) a = 1 / 4
E0 a 2 2
(a) 2 E0 a 2 (b) 2 E0 a 2 (c) E0 a 2 (d) (c) a = 2−1 / 4 R (d) a = R / 3
2
Electrostatics 393

8. Charge is distributed within a sphere of radius R with a 11. A parallel plate capacitor is made of two square plates of side
− 2r ‘a’ separated by a distance d (d<<a). The lower triangular
A
volume charge density ρ( r ) = e a , where A and a are portions is filled with a dielectric of dielectric constant k, as
r2
shown in the figure. Capacitance of this capacitor is
constants. If Q is the total charge of this charge distribution, (2019 Main)
the radius R is (2019 Main)
 1 
 Q 
(a) a log  Q  (b) a log 1 − 
1 −   2πaA 
 2πaA  d
K
 1 
a  Q  a
(c) log 1 −  (d) log  Q 
2  2πaA  2 1 − 
 2πaA  a

9. Charges –q and +q located at A and B, respectively, constitute Kε 0 a 2


Kε 0 a 2
(a) ln K (b) ln K
an electric dipole. Distance AB = 2a, O is the mid point of the d d ( K − 1)
dipole and OP is perpendicular to AB. A charge Q is placed at Kε 0 a 2 1 . Kε 0 a 2
P, where OP = y and y > > 2a. The charge Q experiences an (c) (d)
2d ( K + 1) 2 d
electrostatic force F. (2019 Main)
P 12. There is uniform spherically symmetric surface charge
density at a distance R0 from the origin. The charge
distribution is initially at rest and starts expanding because of
Q P′ mutual repulsion. The figure that represents best the speed
v[ R ( t )] of the distribution as a function of its instantaneous
O radius R ( t ) is (2019 Main)
A B
–q +q v [R(t)] v [R(t)]

If Q is now moved along the equatorial line to P′ such that (a) (b)
 y y 
OP ′ =   , the force on Q will be close to  >> 2a
 3 3 
Ro Ro
F R(t) R(t)
(a) (b) 3F (c) 9F (d) 27F v [R(t)] v [R(t)]
3
10. Two identical parallel plate capacitors of capacitance C each, vo
have plates of area A, separated by a distance d. The space (d)
(c)
between the plates of the two capacitors, is filled with three
dielectrics of equal thickness and dielectric constants K 1 , K 2
Ro R(t) Ro
and K 3 . R(t)
The first capacitor is filled as shown in Fig. I, and the second
one is filled as shown in Fig. II. If these two modified Objective Questions II (One or more correct option)
capacitors are charged by the same potentialV, the ratio of the 13. For spherical symmetrical charge distribution, variation of
energy stored in the two, would be (E1 refers to capacitor (I) electric potential with distance from centre is given in
and E2 to capacitor (II)) : (2019 Main) diagram. Given that (2006)
V
K1

K2 K1 K2 K3

K3
r
(I) (II) r = R0
(a) E1 = K1K 2K 3
q q
E 2 (K1 + K 2 + K 3 )(K 2K 3 + K 3K1 + K1K 2 ) V = for r ≤ R0 and V = for r ≥ R0
4 π ε 0 R0 4 πε 0 r
(K + K 2 + K 3 )(K 2K 3 + K 3K1 + K1K 2 )
(b) E1 = 1 Then, which option(s) are correct
E2 K1K 2K 3
(a) Total charge within 2R0 is q
9 K1K 2K 3
(c) E1 = (b) Total electrostatic energy for r ≤ R0 is zero
E 2 (K1 + K 2 + K 3 )(K 2K 3 + K 3K1 + K1K 2 )
(c) At r = R0 electric field is discontinuous
(K + K 2 + K 3 )(K 2K 3 + K 3K1 + K1K 2 )
(d) E1 = 1 (d) There will be no charge anywhere except at r = R0
E2 9 K1K 2K 3
394 Electrostatics

14. Which of the following statement(s) is/are correct? (2011) 18. Two non-conducting spheres of radii R1 and R2 and carrying
(a) If the electric field due to a point charge varies as r−2.5 uniform volume charge densities +ρ and −ρ, respectively, are
instead of r−2 , then the Gauss’s law will still be valid placed such that they partially overlap, as shown in the figure.
(b) The Gauss’s law can be used to calculate the field At all points in the overlapping region (2013 Adv.)
distribution around an electric dipole
(c) If the electric field between two point charges is zero +ρ –ρ
somewhere, then the sign of the two charges is the same
(d) The work done by the external force in moving a unit R2
positive charge from point A at potentialV A to point B at R1
potentialVB is (VB − V A )
15. A spherical metal shell A of radius R A and a solid metal (a) the electrostatic field is zero.
sphere B of radius RB (< R A ) are kept far apart and each is (b) the electrostatic potential is constant.
given charge +Q. Now they are connected by a thin metal (c) the electrostatic field is constant in magnitude.
wire. Then (2011) (d) the electrostatic field has same direction.
(a) E inside
A = 0 (b) Q A > Q B 19. A parallel plate capacitor has a dielectric slab of dielectric
σ R constant K between its plates that covers 1/3 of the area of its
(c) A = B (d) E on surface
< EBon surface
σB RA A plates, as shown in the figure. The total capacitance of the
capacitor is C while that of the portion with dielectric in
16. Six point charges are kept at the vertices of a regular hexagon between is C1 . When the capacitor is charged, the plate area
of side L and centre O as shown in the figure. Given that covered by the dielectric gets charge Q1 and the rest of the
1 q
K = , which of the following statements(s) is(are) area gets charge Q2 . The electric field in the dielectric is E1
4 πε 0 L2 and that in the other portion is E2 . Choose the correct
correct. (2012) option/options, ignoring edge effects. (2014 Adv.)
F L E
+q –q Q1 E1
P
A S T
O D
+2q E2
–2q

R Q2
B +q C
–q
E1 E1 1
(a) The electric field at O is 6K along OD (a) =1 (b) =
E2 E2 K
(b) The potential at O is zero
Q 3 C 2+ K
(c) The potential at all points on the line PR is same (c) 1 = (d) =
(d) The potential at all points on the line ST is same Q2 K C1 K
17. A cubical region of side a has its centre at the origin. It 20. Let E1 ( r ), E2 ( r ) and E3 ( r ) be the respective electric fields at
encloses three fixed point charges, − q at ( 0, − a/ 4 , 0 ), + 3q at a distance r from a point charge Q, an infinitely long wire
(0, 0, 0) and − q at ( 0, + a/ 4 , 0 ). Choose the correct option(s). with constant linear charge density λ, and an infinite plane
(2012) with uniform surface charge density σ. If E1 ( r0 ) = E2 ( r0 )
z = E3 ( r0 ) at a given distance r0 , then (2014 Adv.)
λ
a (a) Q = 4σπr0 2
(b) r0 =
2πσ
–q (c) E1 ( r0 / 2 ) = 2 E2 ( r0 / 2 ) (d) E2 ( r0 / 2 ) = 4 E3 ( r0 / 2 )
y
–q 3q 21. A uniformly charged solid sphere of radius R has potentialV0
(measured with respect to ∞) on its surface. For this sphere,
x
3V 5V 3V
the equipotential surfaces with potentials 0 , 0 , 0 and
(a) The net electric flux crossing the plane x = + a/2 is equal 2 4 4
to the net electric flux crossing the plane x = − a/2 V0
have radius R1 , R2 , R3 , and R4 respectively. Then,
(b) The net electric flux crossing the plane y = + a/2 is more 4
than the net electric flux crossing the plane y = − a/2 (a) R1 ≠ 0 and ( R 2 − R 1 ) > ( R 4 − R 3 ) (2015 Main)
(c) The net electric flux crossing the entire region is q/ ε 0 (b) R1 = 0 and R 2 > ( R 4 − R 3 )
(d) The net electric flux crossing the plane z = + a/2 is equal (c) 2R < R 4
to the net electric flux crossing the plane x = + a/2 (d) R1 = 0 and R 2 < ( R 4 − R 3 )
Electrostatics 395

22. A point charge +Q is placed just outside an imaginary +q (0,b)


hemispherical surface of radius R as shown in the figure.
Which of the following statements is/are correct? (2017 Adv.)
+Q

Q1 Q2 Q3 Q4
(–2a,0) (–a,0) (+a,0) (+2a,0)
R

Column I Column II
A. Q1 , Q2 , Q3 , Q4 all positive p. +x
(a) The electric flux passing through the curved surface of the
Q  1 B. Q1 , Q2 positive; Q3 , Q4 negative q. −x
hemisphere is − 1 − .
2ε 0  2 C. Q1 , Q4 positive; Q2 , Q3 negative r. +y
(b) The component of the electric field normal to the flat D. Q1 , Q3 positive; Q2 , Q4 negative s. −y
surface is constant over the surface
Q Codes
(c) Total flux through the curved and the flat surfaces is (a) A-r, B-p, C-s, D-q (b) A-s, B-q, C-r, D-p
ε0
(c) A-r, B-p, C-q, D-s (d) A-s, B-q, C-p, D-r
(d) The circumference of the flat surface is an equipotential
25. The electric field E is measured at a point P ( 0, 0, d )
23. An infinitely long thin non-conducting wire is parallel to the
Z-axis and carries a uniform line charge density λ. It pierces a generated due to various charge distributions and the
thin non-conducting spherical shell of radius R in such a way dependence of E on d is found to be different for different
that the arc PQ subtends an angle 120° at the centre O of the charge distributions. Column I contains different relations
spherical shell, as shown in the figure. The permittivity of between E and d. Column II describes different electric
free space is ε 0 . Which of the following statements is (are) charge distributions, along with their locations. Match the
true? (2018 Adv.)
functions in Column I with the related charge distributions in
Column II.

λ Z Column I Column II
P E is
R A. independent p. A point charge Q at the origin
of d
120°

O
A small dipole with point
1 charges Q at ( 0, 0, l ) and − Q at
B. E ∝ q. ( 0, 0, − 1).
d
Q
(Take, 2l << d )

1 An infinite line charge coincident


C. E ∝ r. with the X -axis, with uniform
(a) The electric flux through the shell is 3 Rλ / ε 0 d 2
linear charge density λ.
(b) The z-component of the electric field is zero at all the
Two infinite wires carrying a
points on the surface of the shell
uniform linear charge density
(c) The electric flux through the shell is 2 Rλ / ε 0 parallel to the
(d) The electric field is normal to the surface of the shell at 1 X - axis. The one along
D. E ∝ s.
all points d3 ( y = 0, z = l ) has a charge
density + λ and the one along
Match the Column ( y = 0, z = − l ) has a charge
24. Four charges Q1 , Q2 , Q3 and Q4 of same magnitude are fixed density – λ. (Take, 2l << d ).
along the x-axis at x = − 2a , − a , + a and +2a respectively. A Infinite plane charge coincident
positive charge q is placed on the positive y-axis at a distance t. with the xy-plane with uniform
surface charge density.
b > 0. Four options of the signs of these charges are given in
Column I. The direction of the forces on the charge q is given (a) A → t; B → r,s; C → p; D → q (2018 Adv.)
in Column II. Match Column I with Column II and select the (b) A → t; B → r; C → p, s; D → q
correct answer using the code given below the lists. (c) A → t; B → r; C → p, q; D → s
(2014 Adv.) (d) A → s; B → q, r; C → p; D → t
396 Electrostatics

z
Integer Answer Type Question
26. A solid sphere of radius R has a charge Q distributed in its
volume with a charge density ρ = kra , where k and a are L √3 a
D C
constants and r is the distance from its centre. If the electric 2
R 1 a
field at r = is times that at r = R, find the value of a. y
2 8 (2009)
A
27. At time t = 0, a battery of 10 V is connected across points A B
and B in the given circuit. If the capacitors have no charge x
initially, at what time (in second) does the voltage across 31. Three identical capacitors
V
them become 4 V? [Take : ln 5 = 1.6, ln 3 = 1.1] (2010) C1 ,C 2 and C 3 have a 0
2MΩ 2 µF capacitance of 10. µF each S2
and they are uncharged C1 C3

A B
initially. They are
connected in a circuit as S1 C2

2 µF
shown in the figure and C1
2MΩ
is then filled completely
28. Four point charges, each of + q, are rigidly fixed at the four with a dielectric material of relative permittivity ε r . The cell
corners of a square planar soap film of side a. The surface electromotive force (emf) V0 = 8V. First the switch S 1 is
tension of the soap film is γ. The system of charges and planar closed while the switch S 2 is kept open. When the capacitor
1/ N C 3 is fully charged, S 1 is opened and S 2 is closed
 q2  simultaneously. When all the capacitors reach equilibrium,
film are in equilibrium, and a = k   , where k is a
γ the charge on C 3 is found to be 5µC. The value of ε r =...
(2018 Adv.)
constant. Then N is (2011)

29. An infinitely long solid cylinder of radius R has a uniform Analytical & Descriptive Questions
R
volume charge density ρ. It has a spherical cavity of radius 32. A rigid insulated wire A
2
frame in the form of a right
with its centre on the axis of the cylinder, as shown in the P α
angled triangle ABC, is set Q
figure. The magnitude of the electric field at the point P, q1 I q2
in a vertical plane as
which is at a distance 2R from the axis of the cylinder, is given 60º
shown in figure. Two 30º
23ρR B C
by the expression . The value of k is beads of equal masses m
16kε 0 (2012) each and carrying charges
q1 and q2 are connected by a cord of length l and can slide
z
without friction on the wires.
Considering the case when the beads are stationary determine
(a) (i) The angle α (1978)
R (ii) The tension in the cord
R/2
P (iii) The normal reaction on the beads
y
2R (b) If the cord is now cut what are the value of the charges for
x which the beads continue to remain stationary?
33. The figure shows two S
identical parallel plate
capacitors A and B
30. An infinitely long uniform line charge distribution of charge connected to a battery with
per unit length λ lies parallel to the y-axis in the y-z plane at the switch S closed. The V A B
switch is now opened and
3
z= a (see figure). If the magnitude of the flux of the the free space between the
2 plates of the capacitors is
electric field through the rectangular surface ABCD lying in filled with a dielectric of
the x- y plane with its centre at the origin is dielectric constant (or relative permittivity) 3. Find the ratio
λL
(ε 0 = permittivity of free space), then the value of n is of the total electrostatic energy stored in both capacitors
nε 0 (2015 Adv.) before and after the introduction of the dielectric. (1983)
Electrostatics 397

34. Two fixed, equal, positive charges, each of magnitude 39. In the given circuit, (1988)
−5
q = 5 × 10 C are located at points A and B separated by a R1 E1

distance of 6 m. An equal and opposite charge moves towards


them along the line COD, the perpendicular bisector of the C
line AB. The moving charge, when reaches the point C at a
distance of 4 m from O, has a kinetic energy of 4 J. Calculate E2 R2 R3
the distance of the farthest point D which the negative charge
will reach before returning towards C. (1985)
E3 R4
A +q
E1 = 3E2 = 2E3 = 6 V and R1 = 2R4 = 6 Ω
−q R3 = 2R2 = 4 Ω, C = 5µF.
D C Find the current in R3 and the energy stored in the
O
capacitor.
40. Two fixed charges − 2Q and Q are located at the points with
B +q
coordinates ( − 3a , 0 ) and ( + 3a , 0 ) respectively in the
x-y plane. (1991)
35. A part of circuit in a steady state along with the currents
(a) Show that all points in the x-y plane where the electric
flowing in the branches, the values of resistances etc, is
potential due to the two charges is zero, lie on a circle.
shown in the figure. Calculate the energy stored in the
Find its radius and the location of its centre.
capacitor C ( 4µF ) . (1986)
(b) Give the expression V(x) at a general point on the x-axis
and sketch the function V(x) on the whole x-axis.
1A (c) If a particle of charge +q starts form rest at the centre of
the circle, show by a short quantitative argument that the
3Ω
4V 3Ω 5Ω particle eventually crosses the circle. Find its speed when
it does so.
2A
41. (a) If the same charge of Q as in part (a) above is given to a
C 4 µF 1Ω
spherical conductor of the same radius R, what will be the
3V 1Ω energy of the system?
2A 2Ω 4Ω (b) A charge of Q is uniformly distributed over a spherical
3Ω volume of radius R. Obtain an expression for the energy
1A of the system.
Assume the earth to be a sphere of uniform mass density.
Calculate this energy, given the product of the mass and
36. Three point charges q, 2q and 8q are to be placed on a 9 cm the radius of the earth to be 2.5 × 1031 kg-m.
long straight line. Find the positions where the charges should (c) What will be the corresponding expression for the
be placed such that the potential energy of this system is energy needed to completely disassemble the planet
minimum. In this situation, what is the electric field at the earth against the gravitational pull amongst its
position of the charge q due to the other two charges ? constituent particles? (1992)
(1987)
42. A circular ring of radius R with uniform positive charge
37. Three particles, each of mass 1 g and carrying a charge q, are density λ per unit length is located in the y-z plane with its
suspended from a common point by insulated massless centre at the origin O. A particle of mass m and positive
strings, each 100 cm long. If the particles are in equilibrium charge q is projected from the point P ( R 3 , 0, 0 ) on the
and are located at the corners of an equilateral triangle of side positive x-axis directly towards O, with an initial speed v.
length 3 cm, calculate the charge q on each particle. Find the smallest (non-zero) value of the speed v such that the
( Take g = 10 m / s 2 ). (1988) particle does not return to P. (1993)

38. Three concentric spherical metallic shells, A, B and C of radii 43. Two parallel plate capacitors A and B have the same
a, b and c ( a < b < c ) have surface charge densities σ, − σ and separation d = 8.85 × 10− 4 m between the plates. The plate
σ respectively. (1990) areas of A and B are 0.04 m2 and 0.02 m2 respectively. A slab
(a) Find the potential of the three shells A, B and C. of dielectric constant (relative permittivity) K = 9 has
(b) If the shells A and C are at the same potential, obtain the dimensions such that it can exactly fill the space between the
relation between the radii a, b and c. plates of capacitor B. (1993)
398 Electrostatics

(b) the amount of electrostatic energy stored in the system


A B A B before and after completion of the circuit.
47. Two isolated metallic solid spheres of radii R and 2R are
charged such that both of these have same charge density σ.
The spheres are located far away from each other and
110 V
connected by a thin conducting wire. Find the new charge
(i) (ii) (iii)
density on the bigger sphere. (1996)
(a) The dielectric slab is placed inside A as shown in figure (i) 48. A leaky parallel plate capacitor is filled completely with a
A is then charged to a potential difference of 110 V. material having dielectric constant K = 5 and electrical
Calculate the capacitance of A and the energy stored in it.
conductivity σ = 7.4 × 10−12 Ω −1 m−1 . If the charge on the
(b) The battery is disconnected and then the dielectric slab is
removed from A. Find the work done by the external capacitor at instant t = 0 is q = 8.85 µC, then calculate the
agency in removing the slab from A. leakage current at the instant t = 12 s. (1997)

(c) The same dielectric slab is now placed inside B, filling it 49. In the circuit shown in figure, the battery is an ideal one, with
completely. The two capacitors A and B are then emf V. The capacitor is initially uncharged. The switch S is
connected as shown in figure (iii). Calculate the energy closed at time t = 0. (1998)
stored in the system. S A R
44. Two square metal plates of side 1 m are kept 0.01 m apart like
a parallel plate capacitor in air in such a way that one of their C
V R
edges is perpendicular to an oil surface in a tank filled with an
insulating oil. The plates are connected to a battery of emf
500 V. The plates are then lowered vertically into the oil at a
R B
speed of 0.001 ms −1 . Calculate the current drawn from the
battery during the process. (Dielectric constant of oil = 11, (a) Find the charge Q on the capacitor at time t.
ε 0 = 8.85 × 10−12 C2 N−1 m−2 ). (1994)
(b) Find the current in AB at time t. What is its limiting value
as t → ∞ ?
45. The capacitance of a parallel plate capacitor with plate area A
and separation d, is C. The space between the plates is filled 50. A conducting sphere S 1 of radius r is attached to an insulating
with two wedges of dielectric constants K 1 and K 2 handle. Another conducting sphere S 2 of radius R is mounted
respectively (figure). Find the capacitance of the resulting on an insulating stand. S 2 is initially uncharged.
capacitor. (1996)
S 1 is given a charge Q, brought into contact with S 2 and
A removed. S 1 is recharged such that the charge on it is again Q
and it is again brought into contact with S 2 and removed. This
K2 procedure is repeated n times. (1998)

d
(a) Find the electrostatic energy of S 2 after n such contacts
K1 with S 1 .
(b) What is the limiting value of this energy as n → ∞?
51. A non-conducting disc of radius a and uniform positive
46. Two capacitors A and B with capacities 3µF and 2µF are surface charge density σ is placed on the ground with its axis
charged to a potential difference of 100 V and 180 V vertical. A particle of mass m and positive charge q is
respectively. The plates of the capacitors are connected as dropped, along the axis of the disc from a height H with zero
shown in the figure with one wire of each capacitor free. The initial velocity. The particle has q / m = 4 ε 0 g / σ. (1999)
upper plate of A is positive and that of B is negative. An (a) Find the value of H if the particle just reaches the disc.
uncharged 2 µF capacitor C with lead wires falls on the free (b) Sketch the potential energy of the particle as a function of
ends to complete the circuit. Calculate (1997) its height and find its equilibrium position.
C 52. Four point charges + 8 µC, − 1 µC, − 1 µC and + 8 µC are
2 µF fixed at the points − 27 / 2 m, − 3 / 2 m, + 3 / 2 m and
+ 27 / 2 m respectively on the y-axis. A particle of mass
+ –
3 µF 2 µF 6 × 10−4 kg and charge + 01 . µC moves along the x-direction.
A 100 V 180 V B
Its speed at x = + ∞ is v0 . Find the least value of v0 for which
the particle will cross the origin. Find also the kinetic energy
of the particle at the origin. Assume that space is gravity free.
(a) the final charge on the three capacitors and (1 / 4 π ε 0 = 9 × 109 Nm2 / C2 ). (2000)
Electrostatics 399

53. A small ball of mass 2 × 10−3 kg having a charge of 1µC is a ( a < d ) from S 1 towards S 2 along a line making an angle
suspended by a string of length 0.8 m. Another identical ball π / 4 with the normal to the plates. (2004)
having the same charge is kept at the point of suspension. 56. A conducting bubble of radius a, thickness t ( t << a ) has
Determine the minimum horizontal velocity which should be potentialV. Now the bubble collapses into a droplet. Find the
imparted to the lower ball, so that it can make complete potential of the droplet. (2005)
revolution. (2001)
57. At t = 0, switch S is closed. The charge on the capacitor is
54. A positive point charge q is fixed at origin. A dipole with a
varying with time as Q = Q0 (1 − e− αt ) . Obtain the value of Q0
dipole moment pis placed along the x-axis far away from the
origin with p pointing along positive x-axis. Find : (a) the and α in the given circuit parameters. (2005)
kinetic energy of the dipole when it reaches a distance d from R1
the origin, and (b) the force experienced by the charge q at
this moment. (2003) S

55. There are two large parallel metallic plates S 1 and S 2 carrying R2
C
surface charge densities σ1 and σ 2 respectively (σ1 > σ 2 )
placed at a distance d apart in vacuum. Find the work done by V
the electric field in moving a point charge q a distance

SECTION-3
Objective Questions I (Only one correct option)
1. An infinite line charge of uniform electric charge density λ
lies along the axis of an electrically conducting infinite
cylindrical shell of radius R. At time t = 0, the space inside the p
cylinder is filled with a material of permittivity ε and
electrical conductivity σ. The electrical conduction in the
material follows Ohm's law. Which one of the following
graphs best describes the subsequent variation of the
magnitude of current density j (t) at any point in the material?
(2016 Adv.)
j(t)
j(t) (a) surface charge density on the inner surface is uniform and
 Q
 
 2
(a) (b) equal to
4 πa 2
(b) electric field outside the shell is the same as that of a point
t
(0, 0) t charge at the centre of the shell
(0, 0)
(c) surface charge density on the outer surface depends on p
j(t) j(t)
(d) surface charge density on the inner surface of the shell is
zero everywhere

(c) (d) Objective Questions II (One or more correct option)


3. Two capacitors C1 and C 2 are charged to 120 V and 200 V
t t respectively. It is found that by connecting them together the
(0, 0) (0, 0)
potential on each one can be made zero. Then (2013 Main)
2. Shown in the figure is a shell made of a conductor. It has inner (a) 5C1 = 3C 2
radius a and outer radius b and carries charge Q. At its centre (b) 3C1 = 5C 2
is a dipole p as shown. (c) 3C1 + 5C 2 = 0
In this case, (2019 Main)
(d) 9C1 = 4 C 2
400 Electrostatics

Passage Based Questions A

Passage

Consider an evacuated cylindrical chamber of height h
HV
having rigid conducting plates at the ends and an insulating +
curved surface as shown in the figure. A number of spherical
balls made of a light weight and soft material and coated with
a conducting material are placed on the bottom plate. The
balls have a radius r << h. Now, a high voltage source (HV)
4. Which one of the following statement is correct? (2016 Adv.)
(a) The balls will execute simple harmonic motion between
connected across the conducting plates such that the bottom
the two plates.
plate is at +V0 and the top plate at −V0 . Due to their conducting (b) The balls will bounce back to the bottom plate carrying
surface, the balls will get charge, will become equipotential the same charge they went up with.
with the plate and are repelled by it. The balls will eventually (c) The balls will stick to the top plate and remain there.
collide with the top plate, where the coefficient of restitution (d) The balls will bounce back to the bottom plate carrying
can be taken to be zero due to te soft nature of the material of the opposite charge they went up with.
the balls. The electric field in the chamber can be considered 5. The average current in the steady state registered by the
to be that of a parallel plate capacitor. Assume that there are ammeter in the circuit will be (2016 Adv.)
no collisions between the balls and the interaction between (a) proportional to V02 . (b) proportional to the potential V0 .
them is negligible. (Ignore gravity) (c) zero (d) proportions to V01 / 2

Answers
Topic 1 112. (a,c,d) 113. (a,c) 114. (b,c) 115. (a,c)
1. True 2. True 3. True 4. True 116. (c,d) 117. (a) 118. (a,d) 119. (b,d)
5. False 6. False 7. (b) 8. B 120. (b,d) 121. (a) 122. (b) 123. (c)
ε 0 AV 2ε 0 AV Q2  3  124. T = 8.79 × 10−4 N and θ = 27° 125. No
9. ,− 10. 180°, 11.  V
d d 16πε 0L2  K + 2 126. VAB = 25 V,VBC = 75 V
q2 Q (R + r)
12. −qEA 13. 9 × 109 2 14. −8$i 15. (c) 127. 128. 0.9 A
L 4 πε 0 (R 2 + r2 )
16. (b) 17. (d) 18. (b) 19. (a)
 1 q2 
20. (a) 21. (b) 22. (c) 23. (b) 129. W = 5.824   130. 2
24. (c) 25. (d) 26. (b) 27. (a)  4 πε 0 a 
28. (d) 29. (d) 30. (d) 31. (d)
Section 2
32. (b) 33. (d) 34. (b) 35. (c)
1. (a) 2. (a) 3. (d) 4. (c)
36. (a) 37. (c) 38. (c) 39. (b)
40. (c) 41. (d) 42. (c) 43. (a) 5. (c) 6. (a) 7. (a) 8. (d)
44. (b) 45. (b) 46. (a) 47. (b) 9. (d) 10. (c) 11. (b) 12. (c)
48. (c) 49. (d) 50. (c) 51. (b) 13. (a,b,c,d) 14. (c,d) 15. (a,b,c,d) 16. (a,b,c)
52. (a) 53. (a) 54. (b) 55. (d) 17. (a, c) 18. (c,d) 19. (c,d) 20. (c)
56. (d) 57. (c) 58. (c) 59. (b) 21. (b,c) 22. (a,d) 23. (a,b) 24. (a)
60. (d) 61. (c) 62. (c) 63. (d) 25. (b) 26. 2 27. 2 28. 3
64. (d) 65. (a) 66. (c) 67. (b)
29. 6 30. 6 31. 1.50
68. (c) 69. (b) 70. (c) 71. (b)
 1  q1q2
72. (d) 73. (a) 74. (d) 75. (d) 32. (a) (i) α = 60° (ii) T =   + mg
 4 πε 0  l 2
76. (d) 77. (d) 78. (c) 79. (d)
80. (d) 81. (b) 82. (d) 83. (c) (iii) N P = 3mg , N Q = mg (b) q1q2 = − (−4 πε 0 )mgl 2
84. (d) 85. (b) 86. (b) 87. (c) 3
88. (d) 89. (a) 90. (a) 91. (d) 33.
5
92. (a) 93. (d) 94. (a) 95. (d)
34. Maximum distance from O = 8.48 m
96. (a) 97. (b) 98. (d) 99. (b)
100. (c) 101. (d) 102. (b) 103. (b) 35. 0.288 mJ
104. (c) 105. (a) 106. (d) 107. (d) 36. (a) Charge q should be at a distance of 3 cm from 2q (b) Electric
108. (b) 109. (a,d) 110. (b,d) 111. (b,d) field = 0
Electrostatics 401

37. 3.17 × 10−9 C 47.


5
σ
6
σ σ a 2
38. (a) VA = (a − b + c), VB =  − b + c , 48. 0.198 µA
ε0 ε0  b  CV
49. (a) Q = (1 − e−2t / 3RC )
σ  a2 b2  2
VC =  − + c (b) a + b = c
ε0  c c  (b) i2 =
V

V −2t / 3RC V
e ,
2R 6R 2R
39. 1.5 A from right to left, 1.44 × 10−5 J
qn2
40. (a) Radius = 4a, Centre = (5a, 0) 50. (a) U n =
8πε 0R
Q  1 2 
(b) Vx = −  for x ≤ 3a, QR   R  
n
 Q 2R
4 πε 0  3a − x 3a + x  (b) U ∞ = Here,qn = 1 −   
8πε 0r 2
r   R + r 
Q  1 2   
Vx =  −  for x > 3a
4 πε 0  x − 3a 3a + x  4
51. (a) H = a (b) H =
a
3 3
Qq
(c) v = 52. (v0 )min = 3 m/s, K = 3 × 10−4 J
8πε 0ma
3 Q2 53. 5.86 m/s
41. (a) U = qp pq $
20 πε 0R 54. (a) KE = (b) F = i
4 πε 0d 2
2πε 0d 3
3 GM 2 Q2
(b) U = − , E = 1.5 × 1032 J (c) U = (σ − σ 2 )qa
5 R 8πε 0R 55. W = 1
2ε 0

42. vmin = 1/ 3
2ε 0m  a
56. V ′ = V  
 3t 
43. (a) C A = 2 × 10−9 F, U A = 1.21 × 10−5 J (b) W = 4.84 × 10−5 J
(c) U = 1.1 × 10−5 J CVR2 R + R2
57. Q0 = ,α = 1
−9 R1 + R2 CR1R2
44. i = 4.43 × 10 A
CK 1K 2 K ε A Section 3
45. C R = ln 2 where, C = 0
K2 − K1 K1 d 1. (d) 2. (b) 3. (b,c) 4. (d)
46. (a) q1 = 90 µC, q2 = 210 µC, q3 = 150 µC (b) (i) U i = 47.4 mJ 5. (a)
(ii) U f = 18 mJ
Hints & Solutions
Section-1 ε 0 AV
Here, q=
2. When X-rays fall on the metal ball, some electrons emit from d
it due to photoelectric effect. The ball thus gets positively 10. Due to electrostatic repulsion the charges will move as
charged and on a positively charged ball an electrostatic farthest as possible and the angle between the two strings will
force in the direction of electric field acts. Due to this force be 180° as shown in figure. Tension in each string will be
ball gets deflected in the direction of electric field. equal to the electrostatic repulsion between the two charges.
Thus,
3. Mass of negatively charged sphere will be slightly more than
Q T O T Q
the mass of positively charged sphere because some Fe Fe
electrons will be given to the negatively charged sphere L L
180°
while some electrons will be taken out from the positively
charged sphere. 1 Q×Q Q2
T = Fe = =
4. Electric field between the plates of capacitor is almost 4 π ε 0 ( 2L ) 16π ε 0 L2
2

uniform. Therefore, force on both the protons will be 11. Total charge will remain unchanged.
identical. It hardly matters whether they are placed near Hence, Q = Q ′ or 3CV = ( KC + 2C ) V ′
positive plate or negative plate.
C KC
+ − + −
5. Motion is simple harmonic only when charge −q is not very
far from centre of the ring on its axis. Otherwise motion is
2C 2C
periodic but not simple harmonic in nature.
+ − + −
6. Electric field lines of force does not represent the path of the
V VN
charged particle but tangent to the path at any point on the
line shows the direction of electric force on it and it is not  3 
∴ V′ =  V
always necessary that motion of the particle is in the  K + 2
direction of force acting on it. Projectile motion or circular
motion is the best example of it. 12. Electric field is constant. Therefore electrostatic force qE is
also constant.
8. Magnitude of electric field is greatest at a point where
electric lines of force are most close to each other. Hence, WFe = F ⋅ d (d = displacement)
= ( qEi$ ) ⋅[ r − r ]
s p
9. In the circuit shown in figure, there is a capacitor between
ε0 A = qE$i ⋅ [( − a$i − b$j )] = − qEa
plates 1 and 2 , the capacity of which is : C1 = and
d
13. Force on −q due to charges at 1 and 4 are equal and opposite.
potential difference between its plates is V. Therefore, charge
ε AV Similarly, forces on −q due to charges at 2 and 5 are also
stored in it is, q = C1V = 0 equal and opposite. Therefore, net force on −q due to charges
d at 1, 2, 4 and 5 is zero. Only unbalanced force is between −q
Since, plate 1 is connected with positive terminal, hence this and +q at 3 which is equal to
charge q will be positive.
1 q2 q2
Plate 4 is making two capacitors, one with 3 and other with 5. ⋅ or 9.0 × 109 (attraction)
−2 ε 0 AV 4 πε 0 L2 L2
Hence, charge on it will be −2q or . Charge on it is
d  ∂V $ ∂V $ ∂V $ 
14. E=− i+ j+ k
negative because this is connected with negative plate. ∂x ∂y ∂ z 
Charges on both sides of the plates are shown below.
V = 4 x2
∂V ∂V ∂V
Therefore, = 8x and = 0=
∂x ∂y ∂z
q −q −q q q −q −q q
$
E = − 8xi
1 2 3 4 5
or E at (1 m, 0, 2 m) is −8$i V/m
15. From conservation of mechanical energy
decrease in kinetic energy = increase in potential energy
Electrostatics 403

1 ( Ze ) ( 2e ) 1 Q1 1 Q2
or = 5 MeV 21. VC 1 = VQ 1 + VQ 2 = +
4 πε 0 rmin 4 πε 0 R 4 π ε0 R 2
= 5 × 1.6 × 10−13 J =
1  Q2 
Q1 + 
4 π ε0 R  2
1 2Ze2
∴ rmin = 1  Q1 
4 πε 0 5 × 1.6 × 10−13 Similarly VC 2 = Q2 + 
4 π ε0 R  2
( 9 × 109 ) ( 2 )( 92 ) (1.6 × 10−19 )2
= 1  1 
5 × 1.6 × 10−13 ∴ ∆V = VC 1 − VC 2 =  (Q1 − Q2 ) − (Q1 − Q2 )
4 πε 0 R  2 
(Z = 92) Q2
+Ze + 2e Q1
R√2
rmin R

= 5.3 × 10−14 m C1 R C2
−12
= 5.3 × 10 cm
i.e. rmin is of the order of 10−12 cm.
Q1 − Q2
∴ Correct option is (c). = ( 2 − 1)
2 ( 4 π ε0 R )
16. Electric potential at any point inside a hollow metallic sphere W = ∆U = q∆V = q (Q1 − Q2 ) ( 2 − 1) / 2 ( 4π ε 0 R )
is constant. Therefore, if potential at surface is 10 V,
potential at centre will also be 10 V. 22. Eq

17. Motion is simple harmonic only if Q is released from a point E


not very far from the origin on x-axis. Otherwise motion is y
periodic but not simple harmonic. E –q

18. Since, q is at the centre of two charges Q and Q, net force on


it is zero, whatever the magnitude and sign of charge x
•••
Q q Q q –q
on it. For the equilibrium of Q, q should be negative because (– d, 0) (d, 0)
other charge Q will repel it, so q should attract it. At the same
time, these attractions and repulsions should be equal. The electric field E at all points on the x-axis will not have
the same direction.
1 QQ 1 Qq
= For − d ≤ x ≤ d, electric field is along positive x-axis while
4 πε 0 r2 4 πε 0 ( r / 2 )2
for all other points it is along negative x-axis.
Q Q
or q= or with sign q=− The electric field E at all points on the y-axis will be
4 4 parallel to the x-axis (i.e. $i).
19. From the principle of generator, potential difference depends The electric potential at the origin due to both the charges
only on the charge on inner sphere. Since, charge on inner is zero, hence, no work is done in bringing a test charge
sphere is unchanged. Therefore, potential difference V will from infinity to the origin.
remain unchanged. Dipole moment is directed from the –q charge to the +q
CC
20. In series, C = 1 2 charge ( i.e. −i$ direction).
C1 + C 2
23. The diagramatic representation of given problem is shown in
figure.
– +
– + Q1 = C1V1 = CV
– +
– +
1 µF – +

10 µF S
+ –
+ –
(10 )(1) 10 + –
∴ C net = = µF + –
10 + 1 11 + – Q2 = C2V2 = (2C) (2V) = 4CV
404 Electrostatics

The net charge shared between the two capacitors is 28. Since, the capacitor plates are directly connected to the
Q ′ = Q2 − Q1 = 4CV − CV = 3CV battery, it will take no time in charging.
C
After closing the switch the two capacitors will have the
same potential, sayV ′.
The net capacitance of the parallel combination of the two R
capacitors will be
C ′ = C1 + C 2 = C + 2C = 3C
V
The potential difference across the capacitors will be
Q ′ 3 CV
V′ = = =V
C′ 3C 29. Potential at origin will be given by
The electrostatic energy of the capacitors will be q 1 1 1 1 
V =  x − 2x + 3x − 4 x +....
1 1
U ′ = C ′V ′ 2 = ( 3C ) V 2 4 π ε0  0 0 0 0 
2 2
q 1  1 1 1  q
3 2 = ⋅ 1 − 2 + 3 − 4 + K = 4 πε x ln ( 2 )
= CV 4 πε 0 x0 0 0
2
24. The magnitude of electric field at a distance 30. Electric field within the plates E = EQ 1 + EQ 2
r from the axis is given as : E

+ Q1 + Q2
E2 E1

1 2
λ 1
E= i.e. E ∝ E = E1 − E2
2π ε 0 r r
Q1 Q2
Here, λ is the charge per unit length of the capacitor. = −
2 Aε 0 2 Aε 0
25. Electric field lines never enter a metallic conductor (E = 0, Q1 − Q2
⇒ E=
inside a conductor) and they fall normally on the surface of a 2 Aε 0
metallic conductor (because whole surface is at same
potential and lines are perpendicular to equipotential ∴ Potential difference between the plates
surface).  Q − Q2 
V A − VB = Ed =  1  d
26. Electrostatic force, Fe = eE (for both the particles)  2 Aε 0 
But acceleration of electron, ae = Fe / me and Q1 − Q2 Q1 − Q2
= =
acceleration of proton, a p = Fe / mp  Aε 0  2C
2 
 d 
1 2 1 2
S = ae t1 = a p t 2
2 2 31. Charges on the capacitors are
t2 ae mp q1 = ( 30 ) ( 2 ) = 60 pC
∴ = =
t1 ap me and q2 = ( 20 ) ( 3 ) = 60 pC or q1 = q2 = q (say)
l =0 l =0 The situation is similar as the two capacitors in series are
27. − ∫ E ⋅ dl = ∫ dV = V (centre) − V (infinity) first charged with a battery of emf 50 V and then
l =∞ l =∞
disconnected.
but V (infinity) = 0
l =0 q q
∴− ∫l = ∞ E ⋅ dl corresponds to potential at centre of ring. + − + −

2 pF 3 pF q = 60 pC q = 60 pC
1 q + − + −
and V (centre) = ⋅
4 πε 0 R V1 = 30 V V2 = 20 V

( 9 × 109 ) (1.11 × 10−10 )


= ≈ 2V 50 V
0.5
Electrostatics 405

∴ When S 3 is closed, R
VVVV
V1 = 30 V V
and V2 = 20 V. A C B
32. Net electrostatic energy of the configuration will be i V i=0
i
 q. q Q. q Q. q  V 2RV
U = K + + . VVVV
 a 2a a  2V
1 2V − V
Here, K = i= =
V
4 πε 0 2R + R 3R
− 2q
Putting U = 0 we get, Q = Potential difference between A and B :
2+ 2
V A − V + V + iR = VB
ε0 A
33. Applying C = , we have V  V
t1 t ∴ VB − V A = iR =   R =
d − t1 − t 2 + + 2  3R  3
K1 K 2
NOTE In this problem charge stored in the capacitor can also be
V
A asked, which is equal to q = C with positive charge on B side
A/2 A/2 3
K1 d/2 K2 d/2 and negative on A side because VB > VA .
+ K d
K3 d/2 K3 d/2 38. ∆U = decrease in potential energy
= Ui − U f
2
In parallel 1 1 V + V2 
= C (V12 + V22 ) − ( 2C )  1 
2 2  2 
ε0 ( A / 2 )
d/2 d/2 1
d − d/2 − d/2 + + = C (V1 − V2 )2
K1 K3 4
NOTE We can also apply the formula of loss of energy under
ε0 ( A / 2 ) Kε A
+ = 0 such situations.
d/2 d/2 d
d − d/2 − d/2 + + Loss =
C1C 2
( V1 − V2 )2
K2 K3 2(C1 + C 2 )
Solving this equation, we get
Here, C1 = C 2 = C
K1K 3 K 2K 3
K = +
K1 + K 3 K2 + K3 39. q Q q q Q q

x = –a x=0 x = +a x = –a x=x x=a


34. Potential decreases in the direction of electric field. Dotted
lines are equipotential lines. Initial position Final position

2KQq K . q. q.
y Ui = +
a 2a
 1 1  K . q. q.
and U f = KQq  +  + 2a
C  a + x a − x
1
A B
X Here, K =
4 πε 0
E ∆U = U f − U i
2KQqx2
or | ∆U | = (for x < < a)
a3
∴ V A = VC ∴ ∆U ∝ x2
and V A > VB
40. Electric field is zero everywhere inside a metal (conductor)
35. Electric lines of force never form a closed loop. i.e. field lines do not enter a metal. Simultaneously these are
36. Due to attraction with positive charge, the negative charge on perpendicular to a metal surface (equipotential surface).
capacitor A will not flow through the switch S. 41. According to option (d) the electric field due to P and S and
37. In steady state condition, no current will flow through the due to Q and T add to zero. While due to U and R will be
capacitor C. Current in the outer circuit, added up. Hence, the correct option is (d).
406 Electrostatics

42. dV = − Edx Substituting this value of τ c in Eq. (i) and solving for t,
VA 2 we get
⇒ ∫ dV = − ∫ 30x dx
2
t = 13.86 s
Vo 0
2
∴ Correct answer is (a).
 x3  Now, q = CV
V A − VO = − 30 ×  
 3 0 47. τ = CR
= − 10 × [ 23 − ( 0 )3 ] τ1 = (C1 + C 2 ) ( R1 + R2 ) = 18 µs
= − 10 × 8 = − 80 V  CC   RR  8 2 8
43. Resultant of 1 µF and 2 µF is 3 µF. Now in series, potential τ 2 =  1 2   1 2  = × = µs
 C1 + C 2   R1 + R2  6 3 9
difference distributes in inverse ratio of capacity.
V3µF C  C   RR   2
∴ = or V3 µF =  E τ 3 = (C1 + C 2 )  1 2  = ( 6 )   = 4 µs
Vc 3  C + 3  R1 + R2   3

This is also the potential difference across 2 µF. 48. A ≡ ( − a , 0, 0 ), B ≡ ( 0, a , 0 )


∴ Q2 = ( 2 µF )(V2 µF ) y
 
 2cE   2 
or Q2 =   = E
 c + 3  1 + 3  B
 c
–Q
From this expression of Q2 , we can see that Q2 will increase x
with increase in the value of C (but not linearly). Therefore, A
only options (a) and (c) may be correct. Q
z
d  (C + 3 ) − C  6E
Further, (Q2 ) = 2E  2 
= Point charge is moved from A to B
 (C + 3 )  (C + 3 )
2
dC
V A = VB = 0
= Slope of Q2 verus C graph. or ∆V = 0
i.e. slope of Q2 versus C graph decreases with increase in ∴ W = ∆U = q∆V = 0
the value of C. Hence, the correct graph is (a).
or the correct option is (c).
t
E − RC
44. Charging current, I = e 49. Charge will be induced in the conducting sphere, but net
R charge on it will be zero.
 E t ∴ Option (d) is correct.
Taking log on both sides, log I = log   −
 R  RC
50. From Y to X charge flows to plates a and b.
log I versus t graph is a straight line with negative slope ( qa + qb )i = 0, ( qa + qb ) f = 27 µC
and positive intercept.
When R is doubled, magnitude of slope of curve decreases. 3 µF 6 µF
a b
Also at t = 0, the current will be less. Graph Q represents
18 µC 18 µC
the best. Hence, the correct option is (b).
45. All the three plates will produce electric field at P along 3Ω 6Ω
negative z-axis. Hence, vvvvvv vvvvvv
1A
 σ 2σ σ  2σ $
Ep =  + +  ( − k$ ) = − k
 2ε 0 2ε 0 2ε 0  ε0 9V
Initial figure (when switch was open)
∴ Correct answer is (b).
X
46. Given :VC = 3 VR = 3 (V − VC )
Here,V is the applied potential. 9 µC 36 µC

3 3
∴ VC = V or V (1 − e− t / τ c ) = V 3Ω 1A 6Ω
4 4 vvvvvv vvvvvv
Y
1
∴ e− t / τ c = …(i)
4 9V
Here, τ c = CR = 10 s Final figure (when switch is closed)
Electrostatics 407

27 µC charge flows from Y to X . 1 q2 q2 q2


Ui = = =
∴ Correct option is (c). 2 Ci 2 × 2 4
NOTE In the initial figure, both capacitors are in series and total
1 q2 q2 q2
potential difference across them is 9V. Therefore Uf = = =
q net = Cnet Vnet = (2µF )( 9V ) = 18µC 2 C f 2 × 10 20
In the final figure, q2
∴ Energy dissipated = U i − U f =
V3µF = V3Ω = iR 3 = 3V and V6µF = V6Ω = iR 6 5
Now, q = CV  q2 
This energy dissipated  =  is 80% of the initial stored
51. Inside the cavity, field at any point is uniform and non-zero.  5
Therefore, correct option is (b).  q2 
energy  =  .
52. There will be an electric field between two cylinders (using  4
Gauss theorem). This electric field will produce a potential ∴ Correct option is (d).
difference.
56. For inside points ( r ≤ R )
d 
53. After time t, thickeness of liquid will remain  − vt  . 1 q
3  E = 0 and V = constant =
4 πε 0 R
Now, time constant as function of time
For outside points ( r ≥ R )
ε 0 (1) ⋅ R 1 q
τ c = CR = E= .
d d / 3 − vt 4 πε 0 r2
( d − + vt ) +
3 2 1
or E∝ 2
  r
 ε0 A  1 q
 Applying C =  and V=
 t 4 πε 0 r
d−t+ 
 k 1
or V ∝
6 ε0 R r
=
5d + 3vt On the surface ( r = R )
∴ Correct option is (a). V =
1 q

1
.
q
E=
54. 5 4 πε 0 R 4 πε 0 R 2
4
Corresponding to above equations the correct graphs are
3
2 shown in option (d).
1
57. Between 3 µF and 2 µF (in parallel), total charge of 80 µC
will distribute in direct ratio of capacity.
R q3 3
3R =
q2 2
2R

+
80 µC

q1 = Q1 = σ ( 4 πR 2 ) = 4σπR 2 ...(i)
q2 = − q1 = − ( 4 πσR ) 2
+ +
2 µF, q2 q3 , 3 µF
q3 = σ ( 4 π )( 2R )2 = 16σπR 2 – –
∴ Q2 = q2 + q3 = 12σπR 2 ...(ii)
q4 = − q3 = − 16σπR 2
q5 = σ ( 4 π )( 3R )2 = 36σπR 2  3 
q3 =   ( 80 ) = 48 µC
∴ Q3 = q4 + q5 = 20σπR 2
...(iii)  3 + 2
From equations (i), (ii) and (iii), we get 58. + –
Q1 : Q2 : Q3 = 1 : 3 : 5 qE
45°
qE
55. qi = C iV = 2V = q (say) Å
This charge will remain constant after switch is shifted from mg Net force mg
position 1 to position 2.
408 Electrostatics

Net force should be at 45° from vertical. 2


 x3 
qX  X V A − VO = − 30 ×  
∴ qE = mg or = mg Q E =   3 0
d  d
= − 10 × [ 23 − ( 0 )3 ]
mgd (1.67 × 10−27 ) (9.8) (10−2 ) = − 10 × 8 = − 80 V
or X = =
q (1.6 × 10−19 ) 63. Electric field lines originate from positive charge and
≈ 1 × 10−9 V termination negative charge. They cannot form closed loops
and they are smooth curves. Hence, the most appropriate
59. y
answer is (d).
–q/2
F θ θ F 64. C3 d/2

x C4
q a a q
E1
S/ 2
E2
F sin θ F sin θ
+
2F cos θ
E1 S/ 2
Fnet = 2F cosθ
C5
 q
−2kq  
 2 y d
⇒ Fnet = ⋅ C3 C4
( y2 + a 2 )2 y2 + a 2
 q
−2kq   y + –
 2
Fnet = ⇒ F net ∝ − y
( y2 + a 2 )3 / 2 C5
60. s
2 ε0
L L ε s 2 = 2ε 0 s
C1 = 0 , C 3 =
O A B d d d
x dx
2
 Q s
4 ε0
k   dx 4ε s
2L kdQ 2L  L ⇒ C4 = 2 = 0
V = ∫L x
= ∫L x d d
Q 2 L  1 2
=
4 πε 0 L ∫L
  dx
 x 2ε 0
s
Q and C5 = 2 = ε0 s
= [log e x]L2 L d d
4 πε 0 L
C 3C 4 4 ε0 s ε0 s
Q C2 = + C5 = +
= [log e 2L − log e L] C3 + C4 3 d d
4 πε 0 L
7 ε0 s 7 C2 7
Q = = C1 ⇒ =
= ln( 2 ) 3 d 3 C1 3
4 πε 0 L
kQ 1 65. Resultant of 1 µF and 2 µF is 3 µF. Now in series, potential
61. E1 = , where k = difference distributes in inverse ratio of capacity.
R 2
4 πε 0
V3µF C
k ( 2Q ) 2kQ ∴ =
⇒ E2 = ⇒ E2 = Vc 3
R2 R2
 C 
k ( 4Q ) R kQ or V3 µF =  E
⇒ E3 = ⇒ E3 =  C + 3
( 2R )3 2R 2
This is also the potential difference across 2 µF.
∴ E2 > E1 > E3
∴ Q2 = ( 2 µF )(V2 µF )
62. dV = − Edx  
VA 2  2cE   2 
or Q2 =   = E
⇒ ∫ dV = − ∫ 30x dx
2
 c + 3  1 + 3 
Vo 0  c
Electrostatics 409

From this expression of Q2 , we can see that Q2 will increase So, total number of capacitors required
with increase in the value of C (but not linearly). Therefore, = m × n = 8 × 4 = 32
only options (a) and (c) may be correct.
69. In steady state no current flows through the capacitor.
d  (C + 3 ) − C  6E E
Further, (Q2 ) = 2E  2 
= So, the current in the outermost loop, I =
dC  (C + 3 )  (C + 3 )2 r + r2
= Slope of Q2 verus C graph. Q Potential drop across capacitor = Potential drop across r2
i.e. slope of Q2 versus C graph decreases with increase in Er2
= Ir2 =
the value of C. Hence, the correct graph is (a). r + r2
66. Resultant of 3 µF and 9µF is 12µF CEr2
∴ Charge stored in the capacitor, Q = CV =
4 × 12 r + r2
Now, resultant of 4µ F and 12µF = = 3µF
4 + 12 70. qI = ( 4 π a 2 )(σ ), qII = 4 π b 2 ( − σ ), qIII = 4 π c2 (σ )
Q = CV = 3 × 8 = 24µC (on 4µF and 12µF ) VB = (Potential due to I) + (Potential due to II)
Now, this 24µCdistributes in direct ratio of capacity between + (Potential due to III)
3µF and 9µF. Therefore, K 4 πa 2σ K 4 πb 2 K 4 πc2 (σ )
Q 9 µF = 18 µC VB = + ( −σ ) +
b b c
∴ Q 4 µF + Q 9 µF = 24 + 18 = 42µC = Q 1
kQ 9 × 109 × 42 × 10−6 Substituting K = , we get
E= = 4 π ε0
R2 ( 30 )2
σ  a2 − b2 
= 420 N/ C VB =  + c
ε0  b 
67. τ = PE sin θ
σ
In the first case electric field is along x-axis. So angle
–σ
between P and E is θ.
σ
∴ T1 = PE sin θ ...(i)
In the second case electric field is along y -axis. So angle c a
between P and E is ( 90° − θ )
b
∴ T2 = P ( 3E ) sin( 90° − θ ) I

or T2 = 3 PE cosθ ...(ii) II

Given that, T1 = − T2 III


∴ | T1 | or T1 = |− T2 | or T2 71. Q = KC 0V
Equating (i) and (ii), we get
 1
tan θ = 3 or θ = 60° Qin = Q 1− 
 K
68. Let there are n capacitors in a row with m such rows in  5  3
= ( 90 × 10−12 ) ( 20 )   1−  C
parallel.  3  5
n = 1200 × 10−12 C = 1.2 µC
n
72.
m rows –q
n
d
+ − +q
V=1.0 kV
d E2 E1
As voltage is not to exceed 300 V θ2 P θ1
∴ n × 300 > 1000 O θ2 θ1
10 d
E1
⇒ n> or n = 4 E2
3
+q
mC d OP = D
Also, C Eq = = 2µF
n
–q
m
⇒ = 2 ⇒ m= 8 [QC = 1µ F]
n
410 Electrostatics

The given distribution of charges can be shown in the 1⋅ q1 q2


F =
above figure 4 πε 0 ⋅ r2
Electric field at point P, ∴ Net force on charge ‘Q’ placed at origin i.e.
E = E1 cos θ1 + E1 cos θ1 − E2 cos θ 2 − E2 cos θ 2 at x = 0 in accordance with the principle of superposition
= 2E1 cos θ1 − 2E2 cos θ 2 can be given as
2kq 2kq 1 Q×q 1 Q ×Q
= 2 cos θ1 − cos θ 2 Fnet = ⋅ + ⋅
(d + D ) 2
( 2d )2 + D 2 4 πε 0  d  2 4 πε 0 ( d )2
 
D  2
As, cos θ1 = 2
( d + D 2 )1 / 2 Since, it has been given that, Fnet = 0.
Similarly, cos θ 2 =
D 1 Q ×q 1 Q ×Q
1 ⇒ ⋅ + ⋅ =0
[( 2d ) + D 2 ] 2
2 4 πε 0  d  2 4 πε 0 ( d )2
 
= 2kqD[( d 2 + D 2 ]− 3 / 2 − ( 4 d 2 + D 2 )− 3 / 2 ]  2
− 3/ 2 − 3/ 2 1 Q×q 1 Q×Q
2kqD 
Q
d2   4d 2  ⇒ ⋅ =− ⋅ or q = −
=  1 + 2  − 1 + 2   4 πε 0  d  2 4 πε 0 ( d )2 4
D 3  D   D    
   2
As D > > d , then by applying binomial approximation, we
Kq1 q2
get 75. Ui =
2kq  3 d2  3 4d 2   r
= 2 1 − − 1 −  2   9 × 109 × 1 × 10−6 × 1 × 10−6
D  2D 2  2  D   ∴ Ui = = 9J
1 × 10−3
2kq  9d 2  9kqd 2
= 2  2 = Kq1 q2 9 × 109 × (1 × 10−6 )2
D  2D  D4 Uf = = = 1J
rf 9 × 10−3
1
⇒ E∝ 4 Now,
D
1
1 q1 ⇒ Ui − U f = mB vB2
73. E1 = ⋅ 2
4 πE0 r12
1
y ⇒ 9 − 1= × 4 × 10−6 × vB2
2
3 ⇒ vB2 = 4 × 106 ⇒ vB = 2 × 103 ms −1
76. The system of two charges, i.e. + q and − q that are separated
2
by distance d can be considered as a dipole. Thus, the charge
1 Q would be at D distance from the centre of an electric dipole
θ1 θ2
on its axial line.
1 2 3 4 x
−6 So, the total potential energy of the system will be due to
[ 10 × 10 ]
= ( 9 × 109 ) two components.
12 + 32 (1) Potential energy of dipole’s own system
= 9 10 × 102 Kq 1 q2 Kq 2
(PE )1 = =− …(i)
tan θ1 = 3 ⇒ E1 = [ −9$i + 27$j] × 102 V/ m d d

Similarly, E2 = [ 72i$ − 54 $j] × 102 V/ m –q +q


d
E = E + E = [ 63$i − 27$j] × 102 V/ m
1 2
(2) Potential energy of charge Q and dipole system
74. The given condition is shown in the figure given below, KQq
(PE )2 = − ⋅d …(ii)
x=0 D2
+Q q +Q Hence, total potential energy of the system
d/2
Kq 2 KQq
d (PE )total = (PE )1 + (PE )2 = − − ⋅d
d D2
Then, according to the Coulomb’s law, the electrostatic
1  q 2 Qqd 
force between two charges q1 and q2 such that the distance ⇒ (PE )total =−  + 2
between them is ( r ) given as, 4 πε 0  d D 
Electrostatics 411

77. For a positive line charge or charged wire with uniform 79. Electrostatic energy between two charges q1 and q2 such that
density λ, electric field at distance x is the distance between them r is given as
2kλ λ K q1 q2
E= = …(i) U=
x 2πε 0 x r
So, force on charge q which is at a distance r0 due to this In accordance to the principle of superposition, total energy
2kqλ of the charge system as shown in the figure below is
line charge is F = qE = …(ii) [using Eq. (i)]
x Q
Now, work done when charge is pushed by field by a small
displacement dx is
2kqλ √2 a
dW = F ⋅ dx = ⋅ dx [using Eq. (ii)] a
x
∴ Total work done by field of wire in taking charge q from
90°
distance r0 to distance r will be
r r 2kqλ +q a +q
W = ∫ dW = ∫ ⋅ dx
r0 r0 x Kq 2 KQq KQq
U= + +
= 2kqλ[log x]rr0 = 2kqλ (log r − log r0 ) a a 2a
 r It is given that, U = 0
= 2kqλ log  …(iii) Kq  Q
r0 ∴ q+Q+ =0
a  2 
As we know, from work-kinetic energy theorem,
K final − K initial = W − 2×q
⇒ Q=
1 2 r ( 2 + 1)
⇒ mv − 0 = 2kq × log [using Eq. (iii)]
2 r0 80. The four charges are shown in the figure below,
1/ 2
 4 kqλ  r
⇒ v= log 
 m r0 Q (4, 2)
(0, 2) Q
1/ 2
  r √ 20
∴ v ∝  log  r1=2 r 3=
 r0 (0,0) r4 =
√ 20
78. Given, E = ( Ax + B )i$ N-C −1 r2=2
(0, – 2) (4,–2)
The relation between electric field and potential is given as Q Q
dV = − E ⋅ dx
Electric potential at origin (0, 0) due to these charges can
Integrating on both sides within the specified limits, we get be found by scalar addition of electric potentials due to
2 x2
∴ ∫1 dV = V2 − V1 = − ∫x 1
E ⋅ dx each charge.
KQ KQ KQ KQ
x2
∴ V = + + + … (i)
⇒ V1 − V2 = ∫ E⋅ dx r1 r2 r3 r4
x1
1 1 1 1   1
⇒ V = KQ  + + +  = KQ 1 +
5 
x2 x2
2 2 20 20  
= ∫ ( Ax + B )i$ ⋅ ( dxi$ ) = ∫ ( Ax + B ) ⋅ dx
x1 x1 ( 5 + 1)
⇒V = KQ V …(ii)
Here, A = 20 SI unit, B = 10 SI unit, 5
x1 = 1 and x2 = − 5 Now, if another charge Q is placed at origin, then work
−5
done to get the charge at origin
⇒ V1 − V2 = ∫ ( 20x + 10 ) ⋅ dx W = QV …(iii)
1
−5 By putting the value of V from Eq. (ii) in Eq. (iii), we get
 20x2 
= + 10x = 10[ x2 + x]1−5 ( 5 + 1)
 2 1 W = KQ 2 J
5
= 10 [( −5 ) + ( −5 ) − (1) − (1)]
2 2
Q2  1
or W= 1 +  J
= 10( 25 − 5 − 2 ) = 180 V 4 πε 0  5
412 Electrostatics

81. Given charge distribution is shown in the figure below, Kq


Vp =
σc R + x2
2

σb
Given, R = 3a and x = 4 a
σa Kq Kq
P
∴ VP = = … (i)
r 9a + 16a
2 2 5a
a
c At centre, x = 0
b So, potential at centre is
Kq Kq
VC = = … (ii)
R 3a
Given surface charge densities of each shell are same.
Now, energy required to get this charge from x = 4 a to the
∴ σa = σb = σc …(i) centre is
As, surface charge density of shell of radius ‘r’ and having
 Kq Kq 
Q ∆U = q ∆V = q [VC − VP ] = q  −
charge ‘Q’ is given as σ =  3a 5a 
4 π r2
Kq 2  1 1
So, relation (i) can be rewritten as = −
Qa Qb Qc a  3 5
= =
4 πa 2 4 πb 2 4 πc 2 2 Kq 2
∆U = … (iii)
⇒ Qa : Qb : Qc = a 2 : b 2 : c2 15 a
where Qa , Qb and Qc are charges on shell of radius a , b and This energy must be equal to (or less than) the kinetic
c , respectively. energy of the charge, i.e.
Also, Qa + Qb + Qc = Q 1 2 2 Kq 2
mv ≥
a2 2 15 a
Hence, Qa = ⋅Q
a + b 2 + c2
2 So, minimum energy required is
1 2 2 1 q2
b2 mv = × × (put K = 1 / 4 πε 0 )
Qb = ⋅Q 2 15 4 πε 0 a
a 2 + b 2 + c2
∴ Minimum velocity,
c2
Qc = ⋅Q 2 2 q2
a 2 + b 2 + c2 v2 = × ×
m 4 πε 0 15a
As we know for charged spherical shell with charge Q of
radius ‘R’, the potential at a point 2 2q 2
or v= ×
‘P’ at distance r such that r < R is m 4 πε 0 a × 15
kQ
VP = . 83. Using Gauss’ law at outer surface, let charge on dipole is q ,
R
Σq 1
∴ potential at point P at a distance φ= = E ⋅ A or E = Σq
‘r’ = Potential due to Qa + Potential due to Qb + Potential ε0 Aε 0
due to Qc ( +Q + q − q ) Q σ
= = = = constant
=
kQa kQb kQc
+ + Aε 0 Aε 0 ε 0
a b c
84. Initially when uncharged shell encloses charge Q, charge
Substituting the values of Qa , Qb and Qc , we get
distribution due to induction will be as shown,
Q (a + b + c )
V = . +Q
4 πε 0 ( a 2 + b 2 + c2 ) –Q

82. Potential at any point at distance x from the centre of the ring Q
is given by VB VA b
+q a

R R = 3a

C P The potential on surface of inner shell is


x = 4a
kQ k ( −Q ) kQ
VA = + + …(i)
a b b
Electrostatics 413

where, k = proportionality constant. We know that, electric potential energy stored in an electric
Potential on surface of outer shell is dipole kept in uniform electric field is given by the relation
kQ k ( −Q ) kQ U = − p ⋅ E = − PEcosθ
VB = + + …(ii)
b b b = − 10−29 × 1000 × cos 45°
Then, potential difference is
⇒ U ≈ − 7 × 10−27 J
 1 1
∆V AB = V A − VB = kQ  − 
 a b 89. As, dipole moments points in same direction
dA dB
Given, ∆V AB = V
+ − + −
 1 1 A B
So, kQ  −  = V …(iii)
 a b So, potential of both dipoles can be same at some point
Finally after giving charge − 4Q to outer shell, potential between A and B. Let potentials are same at P, distant x
difference will be from B as shown below
A P B
 kQ k ( −4Q )  kQ k ( −4Q )
∆V AB = V A − VB =  +  − + 
 a b   b b 
R–x x
1 1
= kQ  −  =V [from Eq. (iii)] 4 qa 2qa
a b Then, =
(R − x ) 2
( x )2
Hence, we obtain that potential difference does not depend
on the charge of outer sphere, hence potential difference 2 x2 = ( R − x )2
remains same. R
2x = R − x ⇒ x =
85. For a uniformly charged spherical shell, electric potential 2+1
inside it is given by Distance from A is
Vinside = Vsurface = kq / r0 = constant, R 2R
⇒ R − x= R − =
(where r0 = radius of the shell). 2+1 2+1
and electric potential outside the shell at a distance r is
kq 90. Electric field at a distance ‘h’ from the centre of uniformly
Voutside = charged ring of total charge q (say) on its axis is given as,
r
1 qh
⇒ V ∝ 1/ r E= ⋅ 2
4 π ε 0 ( h + R 2 )3 / 2
∴ The given graph represents the variation of r and
potential of a uniformly charged spherical shell. For the magnitude to be maximum,
dE
86. On perpendicular bisector, =0
dh
p
V = 0, E = Differentiating, we get
4 πε 0 d 3
R
and E↑ ↓ p h=
2
87. | P | = 2 p cos 30 91. Given circuit is
3 1µF
= 2ql
2 4µF
−2q 5µF

30° 30° 3µ F

+q +q

= 3 ql
+ –
P = − 3 ql $j 10V
88. Given, E = 1000 V/m In parallel, C eq = 5 + 1 = 6 µF
θ = 45° 6× 4
′ =
and in series, C eq = 2.4 µF
and p = 10−29 C-m 6+ 4
414 Electrostatics

This is equivalent to or C1C 2 = 8 × (C1 + C 2 ) = 8 × 50 µF


2.4 µF = 400 µF [using Eq. (i)] …(iii)
From Eqs. (i) and (iii), we get
C1 = 50 − C 2 and C1C 2 = 400
3 µF ⇒ C 2 ( 50 − C 2 ) = 400 ⇒ 50 C 2 − C 22 = 400
or C 22 − 50C 2 + 400 = 0
+ 50 ± 2500 − 1600 + 50 ± 30
⇒ C2 = =
+ – 2 2
10 V ⇒ C 2 = + 40 µF or + 10 µF
So, potential difference across upper branch =10 V Also, C1 = 50 − C 2 ⇒ C1 = + 10 µF or + 40 µF
Using, Q = C × V , charge delivered to upper branch is Hence, capacitance of two given capacitors
is 10 µF and 40 µF.
Q = C eq
′ ⋅V = 2.4 µF × 10V
= 24 µC 94. When parallel combination is fully charged, charge on the
As we know, in series connection, same charge is shared combination is
by capacitors, so charge on 4 µF capacitor and 6 µF C

capacitor would be same,


i.e., Q4′ µF = 24 µ C
nC
92. When pendulum is oscillating between capacitor plates, it is
subjected to two forces; +

(i) Weight downwards = mg V

(ii) Electrostatic force acting horizontally = qE Q = C eqV = C (1 + n ) V


So, net acceleration of pendulum bob is resultant of When battery is removed and a dielectric slab is placed
accelerations produced by these two perpendicular forces between two plates of first capacitor, then charge on the
system remains same. Now, equivalent capacitance after
insertion of dielectric is

K KC
qE

anet nC
mg
2
 qE 
Net acceleration is, anet = a12 + a22 = g 2 +   C eq = KC + nC = ( n + K ) C
 m
If potential value after insertion of dielectric is V ′, then
So, time period of oscillations of pendulum is charge on system is
l L Q ′ = C eqV ′ = ( n + K ) CV ′
T = 2π = 2π
anet 2
As Q = Q ′, we have
 qE 
g2 +  
 m C (1 + n )V = ( n + K ) CV ′
(1 + n ) V
93. In the given figure, ∴ V′ =
(n + K )
Slope of OA > Slope of OB
Since, we know that, net capacitance of parallel 95. Potential energy stored in a capacitor is
combination > net capacitance of series combination 1 1 Q2
U = QV =
∴ Parallel combination’s capacitance, 2 2C
500 µC 1
C P = C1 + C 2 = = 50 µF ... (i) So, initial energy of the capacitor, U i = Q 2 / C1
10 V 2
Series combination’s capacitance, 1 2
Final energy of the capacitor, U f = Q / C 2
C1C 2 80 µC 2
CS = = = 8 µF … (ii)
C1 + C 2 10 V As we know, work done, W = ∆U = U f − U i
Electrostatics 415

1  1 1 So, by combining both Eqs. (i) and (ii), we get


= Q2  − 
2 C 2 C1  CV
K = …(iii)
ε 0 AE
Here, Q = 5 µC = 5 × 10−6 C,
Given, C = 15pF = 15 × 10−12 F,
C1 = 5 µF = 5 × 10−6 F,
V = 500 V, E = 106 Vm −1 ,
C 2 = 2 µF = 2 × 10−6 F
A = 10−4 m 2
1  1 1 
⇒ ∆U = × ( 5 × 10−6 )2  − −6 
and ε 0 = 8.85 × 10−12 C 2 N −1 m −2
2  2 × 10–6
5 × 10 
Substituting the values in Eq. (iii), we get
1 5 × 5 × 10−12 3 15 × 10−12 × 500
= × −6
× K =
2 10 10 8.85 × 10−12 × 10−4 × 106
25 × 3 −6
= × 10 J = 8.47 ≈ 8.5
20
98.
⇒ ∆U = 3.75 × 10−6 J
C 2 Ceq = 2×2 =1
2+2
∴ Work done in reducing the capacitance from 5 µF to A
2 µF by pulling plates of capacitor apart is 3.75 × 10−6 J. 2 2 1
96. Net value of charge on plates of capacitor after steady state is
reached is 2 Series
2 2 combination
q2 − q1
qnet = Parallel
2 combination
B
where, q2 and q1 are the charges given to plates.
(Note that this formula is valid for any polarity of charge.) Ceq =2+2=4
Here, q2 = 4 µC, q1 = 2µC
∴ Charge of capacitor is We simplify given circuit as
4−2 C 2
q = ∆qnet = = 1µC
2 A
Ceq = 1+1=2
Potential difference between capacitor plates is
2
Q 1 µC
V = = = 1V 1 1
C 1 µF
4
97. As we know, capacitance of a capacitor filled with dielectric
medium, Ceq = 2×4 = 4 B
2+4 3
ε 0 KA
C= …(i)
d
2 2
d
C
A B

E 4/3
C
2×2 + 4 7
Ceq= =
2+2 3 3
A=area
C × 7/ 3 1
So, C AB = = (given)
C + 7/ 3 2
V
7C C 7
⇒ = +
and potential difference between plates is 3 2 6
V  14 − 3 7
E= ⇒  C=
d  6  6
V 7
⇒ d= …(ii) ⇒ C= µF
E 11
416 Electrostatics

99. If Q = charge on each plate, then


– +
ε A C1= 6µF
Q = CV = 0 ⋅ Ed = ε 0 AE – + – +
d 10µF 2µF
Here, A = 1 m 2 , E = 100 N/C – +
2 C2= 4µF
C
and ε 0 = 8.85 × 10−12 ∴ Charge on 6 µF, capacitor
N − m2
 C1  6
So, by substituting given values, we get =  q= × 30 = 18 µC
 C1 + C 2  6+ 4
Q = 8.85 × 10−12 × 1 × 100 = 8.85 × 10−10 C
Since, the charge has been asked on the right plate of the
dq
100. As we know, Current, I = capacitor. Thus, it would be + 18 µC.
dt
103. Energy stored in a charged capacitor is given by
= Slope of q versus t graph
1 1 Q2
= Zero at t = 4s; (as graph is a line parallel to time axis at U= CV 2 = ⋅ … (i)
t = 4s) 2 2 C
Here, C = 12 × 10− 12 F andV = 10 V.
101. In position ‘A’ of switch, we have a capacitor joined with 1
battery. ⇒ U = × 12 × 10− 12 × 100
2
U = 6 × 10− 10 J … (ii)
After insertion of slab, capacitance will be
ε C C ′ = KC and final energy,
1 Q2 1 Q2
U′ = ⋅ =
2 C ′ 2 KC
1 1
⇒U′ = U = × 6 × 10− 10 J … (iv) (Qgiven, K = 6.5)
So, energy stored in position K 6.5
1
U1 = C ε 2 So, energy dissipated in the process will be equal to work
2 done on the slab, i.e.
When switch is turned to position B, we have a charged  1 − 10
∆U = U − U ′ = 1 −  × 6 × 10 J
capacitor joined to an uncharged capacitor.  6.5
5.5
⇒ ∆U = × 6 × 10− 10 J
6.5
++ ++ ≅ 5.08 × 10− 10 J or 508 pJ
C –– –– 3C
104. In the given arrangement, capacitor can be viewed as three-
different capacitors connected in parallel as shown below,
A
Common potential in steady state will be
total charge Cε ε
V = = = K1 K2 K3
total capacity 4C 4
Now, energy stored will be Plate area A
B
2
1 1  ε 1
U 2 = (C eq ) (Vcommon )2 = 4 C ×   = Cε 2 A
2 2  4 8
So, energy dissipated is
Plate
1 2 1 2
∆U = U1 − U 2 = Cε − Cε area A / 3 K
1 K2 K3
2 8
2
3 3  Q 3 Q2
= Cε 2 = C   = ⋅
8 8 C  8 C
B
102. Applying the concept of charge conservation on isolated
plates of 10 µF, 6 µF and 4 µF. Since, 6 µF and 4 µF are in So, equivalent capacity of the system is
parallel, so total charge on this combination will be 30 µC. C eq = C1 + C 2 + C 3
Electrostatics 417

Kε 0 A K 1 ε 0 A / 3 K 2 ε 0 A / 3 K 3 ε 0 A / 3 107. Total charge = 12 − 3 = 9µC


⇒ = + +
d d d d If final charges are q1 and q2
K1 K 2 K 3
⇒ K = + + q1 C1 R1 2
= = =
3 3 3 q2 C 2 R 2 1
Here, K 1 = 10, K 2 = 12 and K 3 = 14
q1 = 6 µC
10 + 12 + 14
So, K = ⇒ K = 12 q2 = 3 µC
3
108. C2 q
md 2 2
105. I = C1=15µF Q
2
C3=8µF
τ = Iα
q3
md 2
dqE sin θ = α V1 V2=20V
2
+N
d/2 F=qE
q2 + q3 = 750
θ Here, q3 = C 3.V2 = 160 µC
d/2 q2 + 160 = 750
–q q2 = 590 µC
F=– qE
109. When dielectric slab is introduced, capacity gets increased
 2qE  while potential difference remains unchanged.
α=  sin θ ∴ V = V0 , C > C 0
 md 
Q = CV
For θ small
∴ Q > Q0
 2qE 
α= θ 1
 md  U = CV 2
2
∴ ω=
2qE ∴ U > U0
md V
E = butV and d both are unchanged.
106. Forces on the bob are as shown below d
Therefore, E = E0
T cosθ Therefore, correct options are (a) and (d).
θ T
θ E 110. Charging battery is removed. Therefore, q = constant
qE Distance between the plates is increased. Therefore,
T sinθ
C decreases.
mg Now, V = q / C , q is constant and C is decreasing.
Therefore, V should increase.
For equilibrium,
1 q2
T cosθ = mg …(i) U= again q is constant and C is decreasing.
2C
and T sin θ = qE …(ii) Therefore U should increase.
Dividing Eq. (ii) by Eq. (i), we get ∴ Correct options are (b) and (d).
qE
tan θ = 111. The discharging current in the circuit is,
mg
+ –
Here, q = 5µC = 5 × 10−6 C, E = 2000 V / m,
m = 2g = 2 × 10−3 kg, g = 10 ms −2
5 × 10−6 × 2000 1
∴ tan θ = = = 0.5 R
2 × 10−3 × 10 2 VVVV
So, the angle made by the string of the pendulum with the i = i0 e− t / CR
vertical is
V
θ = tan −1 ( 0.5) Here, i0 = initial current =
R
418 Electrostatics

Here, V is the potential difference by which capacitor was 114. The magnitude and direction of electric field at different
charged. points are shown in figure. The direction of the electric field
Since, V and R for both the capacitors are same, initial remains the same. Hence, option (b) is correct. Similarly,
discharging current will be same but non-zero. electric lines always flow from higher potential to lower
∴ Correct option is (b). potential, therefore, electric potential increases continuously
as we move from x = 0 to x = 3d .
Further, τ c = CR ⇒ C1 < C 2 or τC 1 < τC 2
q qi qi
or C1 loses its 50% of initial charge sooner than C 2 .
– +
∴ Option (d) is also correct . – +
– +
112. Battery is removed. Therefore, charge stored in the plates – +
will remain constant. – +
ε0 A – +
Q = CV = V or Q = constant. –
d +
– +
Now, dielectric slab is inserted. Therefore, C will increase. – +
New capacity will be, – E0 / k +
ε KA Q V E0 E0
C ′ = KC = 0 ⇒ V′ = = x=0 x=d x = 2d x = 3d
d C′ K
V′ V Option (c) hence is also correct. The variation of electric
and new electric field, E = = field (E) and potential (V) with x will be as shown below.
d K ⋅d
Potential energy stored in the capacitor, E
V
1 ε AV 2
Initially, U i = CV 2 = 0 C
2 2d d 2d 3d
X B
2 A
1 1  Kε A   V  ε AV 2
Finally, Uf = C ′V ′ 2 =  0    = 0
2 2 d  K  2Kd
V0
Work done on the system will be x
O d 2d 3d
ε 0 AV  1 
2
| ∆U | = 1−  115. Let Q be the charge on the ring, the negative charge − q is
2d  K 
released from point P ( 0, 0, z 0 ). The electric field at P due to
∴ Correct options are (a), (c) and (d). the charged ring will be along positive z-axis and its
1 Q magnitude will be
113. Inside the sphere E = ⋅ 3r
4 πε 0 R y

⇒ E ∝ r for r ≤ R Q
x
1 Q E
i.e. E at centre = 0 as r = 0 and E at surface = ⋅
4 πε 0 R 2 −q
O z
P(0, 0, z0)
as r= R R
Outside the sphere
1 Q Fe
E= ⋅ for r ≥ R
4 πε 0 r2
1
or E∝ E=
1 Qz 0
r2 4 πε 0 ( R + z 02 )3 / 2
2

Variation of electric field (E) with distance (r) from the


centre is as shown below : E = 0, at centre of the ring because z 0 = 0
Force on charge at P will be towards centre as shown, and its
E
magnitude is
1 Qq
Fe = qE = ⋅ ⋅ z0 …(i)
4 πε 0 ( R 2 + z 02 )3 / 2
E= 1 Q
4πε0 R 2 1 Similarly, when it crosses the origin, the force is again
E∝
r2
r

towards centre O.
E∝

r Thus, the motion of the particle is periodic for all values of z 0


O r=R lying between 0 and ∞.
Electrostatics 419

Secondly, if z 0 < < R, ( R 2 + z 02 )3 / 2 ≈ R 3 120. After pressing S 1 charge on upper plate of C1 is + 2CV0 .
1 Qq After pressing S 2 this charge equally distributes in two
Fe ≈ ⋅ ⋅ z0 [From Eq. (i)]
4 πε 0 R 3 capacitors. Therefore, charge on upper plates of both
capacitors will be + CV0 .
i.e. the restoring force Fe ∝ − z 0 . Hence, the motion of the
When S 2 is released and S 3 is pressed, charge on upper
particle will be simple harmonic. (Here negative sign
plate of C1 remains unchanged ( = + CV0 ) but charge on
implies that the force is towards its mean position O, where
upper plate of C 2 is according to new battery ( = − CV0 ).
E is zero, so force on −q is zero.)
121. At r = R , from Gauss’s law
116. Under electrostatic conditions, all points lying on the
qin Ze
conductor are at same potential. Therefore, potential at E ( 4 πR 2 ) = =
A = potential at B. Hence, option (c) is correct. From Gauss ε0 ε0
theorem, total flux through the surface of the cavity will be 1 Ze
or E= ⋅
q / ε0 . 4 πε 0 R 2
NOTE Instead of an elliptical cavity, if it would have been a E is independent of a.
spherical cavity then options (a) and (b) were also correct. ∴ Correct option is (a).
117. Net torque on ( − q ) about a point (say P) lying over + Q is 122. For a = 0
zero. Therefore, angular momentum of ( − q ) about point P  d 
should remain constant. ⇒ ρ( r ) =  − ⋅ r + d 
 R 
118. From the behaviour of electric lines, we can say that Q1 is R  d 
∫0 ( 4 πr )  d − r dr = net charge = Ze.
2
positive and Q2 is negative. Further, |Q1 | > |Q2 | Now
 R 
At some finite distance to the right of Q2 , electric field will be
zero. Because electric field due to Q1 is towards right (away ρ
from Q1 ) and due to Q2 is towards left (towards Q2 ). Since d
magnitude of Q1 is more, the two fields may cancel each other
because distance of that point from Q1 will also be more.
∴ The correct options are (a) and (d).
119. At point P
r
R

Q C1 P C2 3Ze
Solving this equation, we get d =
2R 2R πR 3
123. In case of solid sphere of charge of uniform volume charge
density
If resultant electric field is zero, then ρ(r)
KQ1 KQ2
2
= R
( 2R ) ( 2R )3
4 
Here, Q1 = ρ1  πR 3 
3 
r
R
4 
and Q2 = ρ2  π ( 2R )3
3  E=
1

q
⋅r
Substituting the values, we get 4 πε 0 R 3
ρ1 or E ∝r
=4
ρ2 Thus, for E to be linearly dependent on r, volume charge
At point Q density should be constant.
or a = R.
If resultant electric field is zero then
KQ1 KQ2 ∴ Correct option is (c).
+ =0 Analytical Questions
4 R 2 25R 2
Substituting the values of Q1 and Q2 , we get 124. For equilibrium of bob
ρ1 32 T cos θ = mg
=− (ρ1 must be negative)
ρ2 25 and T sin θ = qE
420 Electrostatics

σ1 = σ 2
θ T q1 q2 q1 r2
θ ∴ = ⇒ = 2
4 πr 2
4 πR 2
q2 R
Fe = qE
i.e., charge on them is distributed in above ratio.
r2 R2
mg
or q1 = Q and q2 = Q
r2 + R 2
r2 + R 2
From these two equations we find Potential at centreV = potential due to q1 +
−1  qE 
T = ( mg ) + ( qE )
2 2
and θ = tan   potential due to q2
 mg  1 q1 1 q2
or V = ⋅ + ⋅
Substituting the values we have 4 πε 0 r 4 πε 0 R
T = ( 80 × 10−6 × 9.8 )2 + ( 2 × 10−8 × 20000 )2 Q (R + r )
=
= 8.79 × 10 −4
N 4 π ε 0 ( r2 + R 2 )
 2 × 10−8 × 20000 128. In steady state situation no current will flow through the
θ = tan −1   = 27°
 80 × 10−6 × 9.8  capacitor.
2Ω and 3Ω are in parallel.
125. Force on a charged particle is along the tangent of electric
line of force. A particle does not always move in the direction Therefore, their combined resistance will be
2× 3
of force acting on it. Projective motion and the circular R=
motion are the best examples. 2+ 3
126. Two capacitors of 3 µF each and two capacitors of 1µF each = 1.2 Ω
are in parallel. Net current through the battery
6
Therefore, simplified circuit can be drawn as below. i= = 1.5 A
6 µF 2 µF 1.2 + 2.8
B
A C This current will distribute in inverse ratio of their
resistances in 2Ω and 3Ω.
1 µF i2 3
∴ =
i3 2

20 Ω 10 Ω  3 
or i2 =   (1.5) = 0.9A
3+ 2 
100 V 129. For potential energy of the system of charges, total number
In steady state no current will flow in the circuit and of charge pairs will be 8 C 2 or 28. Of these 28 pairs, 12 unlike
capacitors are fully charged. charges are at a separation a, 12 like charges are at separation
Between points A and C, 6 µF and 2 µF are in series. 100 V is 2 a and 4 unlike charges are at separation 3 a. Therefore,
applied across this series combination. In series, potential the potential energy of the system is given as
drops in inverse ratio of capacity.
1  (12 )( q )( − q ) (12 )( q )( q ) ( 4 )( q )( − q )
 2  U= + +
∴ V AB = V6 µF =   × 100 = 25 V 4 πε 0  a 2a 3a 
 6 + 2
 1 q2 
 6  = − 5.824  ⋅ 
VBC = V2 µF =  × 100 = 75 V  4 πε 0 a 
 6 + 2
The binding energy of this system is therefore,
127. Let q1 and q2 be the charges on them.
 1 q2 
| U | = 5.824  .
q1 q2  4 πε 0 a 
So, work done by external forces in disassembling, this
r system of charges is
R  1 q2 
W = 5.824  ⋅ 
 4 πε 0 a 
Electrostatics 421

Alternate solution : 3. qE = mg …(i)


W = ∆U = U f − U i = − U i (as U f = 0) 6πη rv = mg
F qE 4 3
130. a= = = 10 sin(10 t )
3 3
πr ρg = mg …(ii)
m m 3
v t
dv
= 103 sin(103 t ) ⇒ ∫ dv = ∫ 103 sin(103 t ) dt
1/ 3
 3mg 
dt ∴ r=   …(iii)
0 0  4 πρg 
3
10
∴ v= [1 − cos (103 t )] Substituting the value of r in Eq. (ii) we get,
103 1/ 3
Velocity will be maximum when cos(103 t ) = − 1  3mg 
6πηv   = mg
 4 π ρg 
v max = 2 m/s
 3 mg 
or ( 6πηv )3   = ( mg )
3

Section-2  4 πρg 
1. Total enclosed charge as already shown is Again substituting mg = qE we get,
6C 8C  3 
q net = + − 7C = − 2C ( qE )2 =   ( 6πηv )
3
2 4  4 π ρg 
q −2C 1/ 2
From Gauss-theorem, net flux, φ net = net =  3 
ε0 ε0 or qE =   ( 6πηv )3 / 2
 4 πρg 
2. Pressure or force per unit area is given by, 1/ 2
1  3 
2 ∴ q=   ( 6πηv )3 / 2
1 1 σ σ 2
E  4 πρg 
P = ε0 E 2 = ε0   =
2 2  ε0  2ε 0
Substituting the given values we get ,
Therefore small electrostatic force on small area will be q = 8.0 × 10−19 C
given by,
 σ2  4. Electric flux, φ = E ⋅ S or φ = ES cosθ
dFe = ( P )( dS ) =   ds Here, θ is the angle between E and S.
 2ε 0 
In this question θ = 45°, because S is perpendicular to
dFe surface.
y E = E0
S = ( 2a ) ( a ) = 2a 2
πR2 F x
∴ φ = ( E0 ) ( 2a 2 ) cos 45° = E0 a 2
dFe ∴ Correct option is (c)
ds is also in the direction of dFe . We can see that 5. At centre both charges are in equilibrium, +q charge is
y -components of dFe are cancelled only x -components are suppose displaced towards right, then repulsion on it will
left. increase from right hand side line charge. Hence it starts
 σ 2   oscillations. These small oscillations come out to be simple
∴ Fe = Σ ( dFe )x = Σ   ds harmonic in nature. In case of −q charge attraction from right
 2ε 0   x hand side line charge will increase. So, net force is towards
right in the direction of its displacement.
σ2
= Σ ( ds )x
2ε 0 6. As E is constant, Hence, Ea = Eb
Now, Σ ( ds )x = projected area = πR 2 According to Guass theorem, only Qin contributes in
electric field.
 σ2 
∴ Fe =   ( πR )
2
towards right
 2ε 0 
 σ2  dr
Hence, external force F is also   ( πR 2 ) towards left Q
 2ε 0  r
a b
for equilibrium of hemispherical shell.
σ 2R 2
or F ∝
ε0
422 Electrostatics

 b A  r4 
R
k Q + ∫ 4 πr2 dr ⋅  2Q
⇒ 2Q = 4 πk   ⇒ k =
= 
kQ a r
∴ 2 2  4 0 πR 4
a b
1 Now, using Gauss’ law, electric field on the surface of
Here, k= sphere of radius a is
4 πε 0
a a
 r2 b  1
E ∫ dA =
ε 0 ∫0
b2 ⋅ ( kr 4 πr2 dr )
⇒ Q 2 = Q + 4 πA  
a  2 a 0
 
1  a4 
 b2 − a2  ⇒ E ⋅ 4 πa 2 = ⋅ 4 πk  
= Q + 4 πA ⋅   ε0  4
 2 
ka 2 2Q a 2
 b2  ⇒ E= =
⇒ Q  2  = Q + 2πA ( b 2 − a 2 ) 4ε 0 4 πε 0 R 4
a 
Force of attraction on charge A (or B ) due to this field is
 b2 − a2 
⇒ Q  = 2πA ( b − a )
2 2 2Q 2 a 2
 a 2
 F1 = QE =
4 πε 0 R 4
Q
⇒ A= Force of repulsion on charge A due to B is
2πa 2
1 Q2 1 Q2
7. F2 = = ⋅
+ +
+ + + 4 πε 0 ( 2a )2 4 πε 0 4 a 2
+ + + + + +
+ + + + 2Q, R If charge A(or B) does not feel any force, then
+ + + +
++ + + +
+
+ ρ(r)=kr F1 = F2
+ + + + + +
A a C + B + 2Q 2 a 2 1 Q2
+ + ⇒ = ⋅ 2
+ + + a + 4 πε 0 R 4 4 πε 0 4 a
+ + + + + ++
+ + + + + ++ ⇒ 8a 4 = R 4
+ + +
a = 8−1 / 4 R
+
+ + ⇒
+ + +
8. Here, volume charge density,
2r
A −
Key Idea Force on A is zero only when repulsion of A and B = ρ( r ) = 2
⋅e a
attraction of positive charge distribution of radius a and charge A. r
In given charge distribution, let r is radius of a shell of dr
thickness dr.

r
dr
O
R
R

r
ρ(r)=kr

where, a and A are constant.


2Q

Let a spherical region of small element of radius r.


If Q is total charge distribution upto radius R, then
R R
Charge dQ present in shell of thickness dr = Volume of Q = ∫ ρ ⋅ dV =
A
∫ r2 e
−2 r / a
( 4 πr2 dr )
shell × Volumetric charge density 0 0
⇒ dQ = ( 4 πr2 × dr ) × ( kr ) (From figure, we observe dV = A ⋅ dr = 4 πr2 ⋅ dr)
= 4 πkr3 dr R
R –2 r / a  e −2 r / a 
Total charge in sphere is = 4 πA ∫0
e dr = 4 πA 
 −2 / a  0

R R
2Q = ∫ dQ = ∫ 4 πkr3 dr  − a
= 4 πA ×   ( e−2 R / a − e0 )
0 0  2
Electrostatics 423

= 2πA ( − a ) [ e−2 R / a − 1] From Eqs. (iii) and (iv), we get


F ′ = 27 F
or Q = 2πa A (1 − e−2 R / a )
1
  10. E = CV 2
  2
a 1
or R = log   ⇒ E ∝C
2 1 − Q  E1 C1
 2πaA  =
E2 C 2
9. Electric field on the equatorial line of a dipole at any point,
ε0 A
which is at distance r from the centre is given by Here, C1 =
d d d d/3 d/3 d/3
2kP d− − − + + +
E= 2 … (i) 3 3 3 K1 K2 K3
( r + a 2 )3 / 2
K1 ε0 ( A / 3 ) K 2 ε0 ( A / 3 ) K 3 ε0 ( A / 3 )
where, P is the dipole moment of the charges. and C2 = + +
d d d
y d
P 11. =
x a
y d
P′ ⇒ y= x ...(i)
y/3 a
A B
O ε 0 adx
–q +q
∫ dC = ∫ y
...(ii)
d − y+
K

In first case r= y d
2kP y
⇒ E1 =
( y2 + a 2 )3 / 2 x
dx
Here, y >> a
2 2
Substituting Eqs. (i) and (ii), we get
⇒ y2 + a 2 ≈ y2 a
dx
C = ε0 a ∫
E1 =
2kP
0 d +
dx 1 
or … (ii)  − 1
y3 a K 
So, force on the charge in its position at P will be ε0 a 2 K
2kPQ ⇒ C= ln K
F = QE1 = … (iii) d ( K − 1)
y3
In second case r = y / 3 KQ 2 KQ 2 1 2
12. = + mv
From Eq. (i), electric field at point P′ will be 2r0 2r 2

E2 =
2kP KQ 2  1 1
3/ 2 ⇒ v=  r − r
 y  2  m 0 
  + a 
2
 3
  dv
is positive and decreasing
y dr
Again, > > a ∴ Graph will be best represented by option (c).
3
2 2
 y  y 13. The given graph is of charged conducting sphere of radius
⇒   +a ≈ 
2
 3  3 R0 . The whole charge q is distributed on the surface of the
2kP sphere.
⇒ E2 =
( y / 3 )3 14. If charges are of opposite signs then the two fields are along
the same direction. So, they cannot be zero. Hence, the
2kP
⇒ E2 = 27 × 3
charges should be of same sign.
y
Therefore, option (c) is correct.
Force on charge in this position,
Further, Work done by external force = change in potential
2kPQ energy
F ′ = QE2 = 27 × … (iv)
y3 ∴ W A → B = q ( ∆V ) = ( +1) (VB − V A )
424 Electrostatics

or W A → B = VB − V A Option (c) is correct because as per Gauss’s theorem, net


q
Therefore, option (d) is also correct. electric flux passing through any closed surface = in
∴ Correct options are (c) and (d). ε0
NOTE For option (a), students are advised to see the proof of Here, qin = 3q − q − q = q
Gauss theorem.
q
15. Inside a conducting shell electric field is always zero. ∴ Net electric flux =
ε0
Therefore, option (a) is correct. When the two are connected,
Option (d) is wrong because there is no symmetry in two
their potentials become the same.
given planes.
∴ V A = VB
NOTE Since centre of cube is at origin and side of cube is a.
Q A QB  1 Q Therefore six faces of the cube are :
or = V = 
R A RB  4 πε 0 R  a a
x = ± , y = ± and z = ±
a
2 2 2
Since, R A > RB
18.
∴ Q A > QB
∴ Option (b) is correct.
ρ –ρ
σR P
Potential is also equal to, V = , V A = VB
ε0 C1 C2
R2
∴ σ A R A = σ B RB R1

σA R
or = B  q 
σB RA 1 q 1   r
Ei = ⋅ 3r= 4
σ A < σB 4 πε 0 R 3ε 0  πR 3 
or 3 
∴ Option (c) is correct. ρ
= r
σ 3ε 0
Electric field on surface, E = or E ∝ σ
ε0 ρ (− ρ )
∴ EP = E1 + E2 = C1 P + C2 P
Since, σ A < σ B ∴ E A < EB 3ε 0 3ε 0
∴ Option (d) is also correct. ρ
= (C1 P + PC2 )
∴ Correct options are (a), (b), (c) and (d). 3ε 0
ρ
EP = C1C2
16. (a) +q E
3ε 0
–q
F ∴ EP = constant
EE = K
EB = K, 19. C = C1 + C 2
2K
O EE = K Kε 0 A / 3
4K +2q D C1 =
120° A EA = 2K –2q d
4 K + 2K
2K = 6K
ED = 2K ε 2A / 3
EC = K
C2 = 0
d
EF = K
+q –q ( K + 2 )ε 0 A
B C ⇒ C=
3d
Resultant of 2K and 2K (at 120°) is also 2K towards C K +2
4K .Therefore, net electric field is 6K . ⇒ =
C1 K
1  q A qB qC q q q 
(b) V0 = + + + D + E + F Also, E1 = E2 = V / d , whereV is potential difference
4 πε 0  L L L L L L
between the plates.
1
= ( q A + ... + qF ) = 0 Q λ σ
4 πε 0 L 20. = =
4 πε 0 r02 2πε 0 r0 2ε 0
Because q A + qB + qC + qD + qE + qF = 0
(c) Along line PR, potential is same ( = 0 ) . ⇒ Q = 2πσr02
λ
17. Option (a) is correct due to symmetry. (a) is incorrect, r0 =
πσ
Option (b) is wrong again due to symmetry.
Electrostatics 425

r 
(b) is incorrect, E1  0  = 4 E1 ( r0 )
 2
1
As E1 ∝ 2
r
45°
r  1
E2  0  = 2E2 ( r0 ) as E2 ∝
 2 r R
(c) is correct D
r  R
E3  0  = E3 ( r0 ) = E2 ( r0 ) A B
 2 E
C
as E3 ∝ r0 ⇒ (d) option is incorrect
Area of spherical surface ACB
Kq
21. V0 = potential on the surface = S = 2πR 2 (1 − cos θ )
R
1 where θ = 45°
where, K = and q is total charge on sphere.
4 πε 0  1
∴ S = 2πR 2 1 − 
3 Kq 3  2
Potential at centre = = V0
2 R 2 Now flux passing through AEBD and ACB will be
Hence, R1 = 0 same.
3 Q
From centre to surface potential varies between V0 and V0 Flux passing through whole sphere is . Therefore flux
2 ε0
5V
From surface to infinity, it varies betweenV0 and 0, 0 will passing through surface ACB would be,
4
 (Q / ε 0 )   1 
φ= 2 
2πR 2 1 − 
2  
be potential at a point between centre and surface. At any
 4 π R  
point, at a distance r( r ≤ R ) from centre potential is given by
Kq  3 2 1 2  Q  1
V = = 1 − 
 R − r  2ε 0  2
R3  2 2 
NOTE There is no proper logic of negative sign but IIT website
V0  3 2 1 2 
=  R − r  has given (a) option also correct.
R2  2 2 
(b) The component of the electric field perpendicular to the
5
PuttingV = V0 and r = R2 in this equation, we get flat surface will decrease as we move away from the
4 centre as the distance increases (magnitude of electric
R field decreases) as well as the angle between the normal
R2 =
2 and electric field will increase. Hence, the component of
the electric field normal to the flat surface is not
3V0 V
and 0 are the potentials lying betweenV0 and zero hence constant.
4 4
Q
these potentials lie outside the sphere. At a distance r(≥ R ) (c) Total flux φ due to charge Q is ⋅
Kq V0 R ε0
from centre potential is given by V = =
r r
3 4
PuttingV = V0 and r = R3 in this equation we get, R3 = R 45°
4 3
V0
Further putting V = and r = R4 in above equation,
4
we get R4 = 4 R
Thus, So, φ through the curved and flat surface will be less
R 4R Q
R1 = 0, R2 = , R3 = than .
2 3 ε0
and R4 = 4 R with these values, options (b) and (c) are (d) Since, the circumference is equidistant from Q it will be
correct. KQ
equipotential V = .
22. (a) Flux passing through curved surface and flat surface 2R
will be same and flux passing through flat surface can 1
where, k =
be obtained by the method discussed below. 4 πε 0
426 Electrostatics

23. PQ = ( 2 ) R sin 60° (D) Component of forces along y-axis are cancelled and net
force is along negative x-axis
λ Z
P +q
R
120° O

R
Q +Q –Q +Q –Q

25. List-II
1 Q
3 (p) E =
= ( 2R ) = ( 3R ) 4 πε 0 d 2
2
qenclosed = λ ( 3R ) 1
⇒ E∝
q d2
We have, φ = enclosed 1 2Q ( 2l )
ε0 (q) Eaxis =
4 πε 0 d 3
 3 λR 
⇒ φ=  1
 ε0  ⇒ E∝
d3
Also, electric field is perpendicular to wire, so λ 1
(r) E = ⇒ E∝
Z-component will be zero. 2πε 0 d d
24. (A) Component of forces along x-axis are cancelled. So net λ λ λ ( 2l )
(s) E = − =
force is along positive y -axis. 2πε 0 ( d − l ) 2πε 0 ( d + l ) 2πε 0 d 2
E3 < E1 < E2 1
⇒ E∝ 2
d
+q σ
(t) E = ⇒ E is independent of d
2 ε0
26. From Gauss theorem,
q
E∝ (q = charge enclosed)
r2
+Q +Q +Q +Q
E2 q2 r12
(B) Component of forces along y-axis are cancelled and net ∴ = ×
E1 q1 r22
force is along positive x-axis R

∫ ( 4 πr
2
)kra dr
+q ( R / 2 )2
or 8= 0
R/2
×
( R )2
∫ ( 4 πr )kr dr
2 a

Solving this equation we get,


a = 2.
+Q +Q –Q –Q
27. Voltage across the capacitors will increase from 0 to 10 V
(C) Component of forces along x-axis are cancelled and net exponentially. The voltage at time t will be given by
force is along negative y-axis V = 10 (1 − e− t / τ C )
+q Here, τ c = C net Rnet
= (1 × 106 ) ( 4 × 10−6 ) = 4 s
∴ V = 10 (1 − e− t / 4 )
Substituting V = 4 volt, we have
+Q –Q –Q +Q 4 = 10 (1 − e− t / 4 )
Electrostatics 427

3 Substituting the values, we have


or e− t / 4 = 0.6 =
5 ( πR 2ρ ) 1 ( πR 3 ρ/ 6 )
ER = − .
Taking log on both sides we have, 4 πε 0 R 4 πε 0 4R 2
t 23ρR 23ρR
− = ln 3 − ln 5 = =
4 96ε 0 (16 ) ( 6 ) ε 0
or t = 4 (ln 5 − ln 3 ) = 2 s
∴ k=6
Hence, the answer is 2 .
30. ANBP is cross-section of a cylinder of length L. The line
28. q q
charge passes through the centreO and perpendicular to paper.
P
a

q q
O
Let us make free body diagram of any half portion. 30° 30°
F1 A M B
F2
N
a
a 3a
F3 AM = , MO =
2 2
F2
 AM  −1  1 
∴ ∠AOM = tan −1   = tan   = 30°
For equilibrium,  OM   3
1 q2  1 Electric flux passing from the whole cylinder
electrostatic force F1 =  2+  should be equal
4 πε 0 a 2  2 q λL
φ1 = in =
to, ε0 ε0
surface tension force F3 = 2 ( γ ) ( 2a ) ∴ Electric flux passing through ABCD plane surface (shown
Equating F1 and F3 , we get only AB) = Electric flux passing through cylindrical surface
 q2  ANB
a 3 = k1  
 γ  60°  λL
=  (φ ) =
1  360°  1 6 ε0
 q2  3
∴ a = k  ∴ n=6
γ 31.
Here k1 and k are constants. So, the answer is N = 3.
+8µC
29. Volume of cylinder per unit length ( l = 1) is V0=8V Before 1µF
–8µC
V = πR 2 l = ( πR 2 )
∴ Charge per unit length,
λ = (Volume per unit length) × (Volume charge density)
3µC + C
= ( πR 2ρ ) – +5µC
After 1µF
Now at P –5µC
3µ C
ER = ET − EC 1µF –3µC
R = Remaining portion
T = Total portion and C = ε r C1 = ( ε r ) µF
C = cavity Applying loop rule,
λ 1 Q 5 3 3
− − =0
∴ ER = −
2πε 0 ( 2R ) 4 πε 0 ( 2R )2 1 εr 1
Q = charge on sphere 3
⇒ =2
3 εr
4  R πR 3ρ
= π   ρ= ε r = 150
.
3  2 6
428 Electrostatics

32. Tension and electrostatic force are in opposite directions and 3CV 2 CV 2 10 2 5CV 2
∴ U Total = + = CV = = Uf …(ii)
along the string. Now each bead is in equilibrium under three 2 6 6 3
concurrent forces Ui 3
(i) Normal reaction ( N ) From Eqs. (i) and (ii), we get =
Uf 5
(ii) Weight (mg) and
34. Equating the energy of ( −q ) at C and D
1 qq
(iii) T − Fe , where Fe = ⋅ 12 2 K C + UC = K D + U D ...(i)
4 πε 0 l
Here, KC = 4 J
Applying Lami’s theorem for both beads.
 1 ( q )( − q )
UC = 2  
 4 πε 0 AC 
N1 90°
− 2 × 9 × 109 × ( 5 × 10−5 )2
α = = − 9J
60° 90° – N2 5
α
T – Fe 30° KD = 0
mg
mg  1 ( q )( − q )
30° 60° and UD = 2 
 4 πε 0 AD 
N1 mg
= − 2 × 9 × 109 × ( 5 × 10−5 )2
sin (120° − α ) sin ( 90° + α ) or cos α =
AD
T − Fe
= …(i) 45
sin ( 90° + 60° ) or cos 60° =−
AD
N2 mg
= Substituting these values in Eq. (i)
sin ( 60° + α ) sin (180° − α ) or sin α
45
T − Fe 4 − 9= 0−
= …(ii) AD
sin( 90° + 30° ) or cos 30°
∴ AD = 9 m
Dividing Eq. (i) by Eq. (ii), we have
∴ OD = AD 2 − OA 2
cos 30°
tan α = = 3
cos 60° = ( 9 )2 − ( 3 )2
⇒ α = 60°
= 81 − 9 = 8.48 m
 1  q1 q2
T = Fe + mg =  ⋅ + mg …(iii)
 4 πε 0  l 2 35. Using Kirchhoff’s first law at junctions a and b, we can find
the currents in other wires of the circuit.
N 1 = 3 mg and N 2 = mg
1A
From Eq. (iii) T = 0 when string is cut.
3Ω
or q1 q2 = − ( 4 πε 0 ) mgl 2 4V 3 Ω 2 Aa 3 A 5 Ω
33. Before opening the switch potential difference across both
the capacitors is V, as they are in parallel. Hence, energy 4 µF 1Ω
C
stored in them is, 3V
1 1Ω 2A 4Ω
U A = U B = CV 2 b 3A 2Ω
2 6A
∴ U Total = CV 2 = U i …(i) 3Ω

After opening the switch, potential difference across A is V 1A


and its capacity is 3C
1 3 Now, to calculate the energy stored in the capacitor we will
∴ U A = ( 3C )V 2 = CV 2 have to first find the potential difference Vab across it.
2 2
Va − 3 × 5 − 3 × 1 + 3 × 2 = Vb
In case of capacitor B, charge stored in it is q = CV and its
∴ Va − Vb = Vab = 12 V
capacity is also 3C.
1 1
q2 CV 2 ∴ U = CVab2 = ( 4 × 10−6 ) (12 )2 J
Therefore, U B = = 2 2
2( 3C ) 6 = 0.288 mJ
Electrostatics 429

36. 2q q 8q ∴ θ ≈ 89°
••• Now, the particle is in equilibrium under three concurrent
x
9− x
forces, F, T and mg. Therefore, applying Lami’s theorem
For potential energy to be minimum the bigger charges
T
should be farthest. Let x be the distance of q from 2q. Then
potential energy of the system shown in the figure would be
θ
 ( 2q )( q ) ( 8q )( q ) ( 2q )( 8q ) F
U = K + + 
 x (9 − x ) 9 
1
Here, K =
4 πε 0 mg
2 8 F mg
For U to be minimum + should be minimum. =
x 9− x sin ( 90° + θ ) sin (180° − θ )
d 2 8  or 3 × 1013 q 2 = (1 × 10−3 )(10 ) cot 89°
 x + 9 − x = 0
dx   Solving this equation, we get
−2 8 q = 0.317 × 10−8 C or q = 3.17 × 10−9 C
∴ + =0
x2 ( 9 − x )2
38. (a) Potential at any shell will be due to all three charges.
x 1 1  q A qB qC 
∴ = VA = + +
9− x 2 4 πε 0  a b c 
or x = 3 cm 1  ( 4 πa 2 ) (σ ) ( 4 πb 2 ) ( −σ ) ( 4 πc2 ) (σ )
i.e. distance of charge q from 2q should be 3 cm. =  + + 
4 πε 0  a b c 
Electric field at q
σ
K ( 2q ) K ( 8q ) = (a − b + c )
E= − =0 ε0
( 3 × 10−2 )2 ( 6 × 10−2 )2
1  q A qB qC 
37. F is the resultant of electrostatic forces between two charges. VB =  b + b + c 
4 πε 0
q
1  ( 4 πa 2 ) (σ ) ( 4 πb 2 ) ( −σ ) ( 4 πc2 ) (σ )
=  + + 
4 πε 0  b b c 
a a

σ  a2 
=  −b+ c
Fe
30 ° q a q ε0  b 
°
F 30
Fe Horizontal plane
σ
 1 q2  3 C
–σ
F = 2Fe cos 30° = 2  ⋅ 2 B σ
A
 4 πε 0 a  2 a

2 × 10 × 9 × q × 3
9 2
= −2 2
= 3 × 1013 q 2 …(i) b c
( 3 × 10 ) × 2
θ is the angle of string with horizontal in equilibrium,
1  q A qB qC 
Similarly, VC = + +
4 πε 0  c c c 
l
1  ( 4 πa 2 ) (σ ) ( 4 πb 2 ) ( −σ ) ( 4 πc2 ) (σ )
θ a r =  + + 
4 πε 0  c c c 
r 30°
a a σ  a2 b2 
2 =  − + c
ε0  c c 
r ( a / 2 ) sec 30° a 3
cos θ = = = = (b) Given V A = VC
l l 3l 100 3
430 Electrostatics

σ σ  a2 b2  V = (potential due to Q) + (potential due to − 2Q)


∴ (a − b + c ) =  − + c
ε0 ε0  c c  1  Q 
or V =  
a2 b2 4 πε 0  ( 3a − x )2 + y2 
∴ a−b+c= − +c  
c c
1  − 2Q 
or a+b=c +   …(i)
4 πε 0  ( 3a + x )2 + y2 
39. In steady state no current will flow through R1 = 6 Ω.  
Potential difference across R3 or 4 Ω is E1 or 6 V Given V = 0
6 ∴ 4[( 3a − x )2 + y2 ] = ( 3a + x )2 + y2
∴ Current through it will be = 1.5A from right to left.
4
On simplifying, we get ( x − 5a )2 + y2 = ( 4 a )2
Because right hand side of this resistance is at higher
potential. This is the equation of a circle of radius 4a and
centre at ( 5a , 0 ).
6V
a 6Ω b (b) On x-axis, potential will be undefined (or say ± ∞) at
vvvvvv c
5 µF
i=0 x = 3a and x = − 3a, because charge Q and − 2Q are
i2 placed at these two points.
2 Ω i1 4Ω
d vvvvvv e vvvvvv f So, between − 3a < x < 3a we can find potential by
1.5 A
2V putting y = 0 in Eq. (i). Therefore,
i1
3Ω Q  1 2 
V = − for − 3a < x < 3a
4 πε 0  3a − x 3a + x
g vvvvvv h
3V
V = 0 at x = a
Now, suppose this 1.5 A distributes in i1 and i2 as shown.
Applying Kirchhoff’s second law in loop dghfed V → − ∞ at x → − 3a
3 − 3i1 − 4 × 1.5 − 2i1 + 2 = 0 and V → + ∞ at x → 3a
1 For x > 3a , there is again a point where potential will
∴ i1 = − A = − 0.2A
5 become zero so for x > 3a, we can write
To find energy stored in capacitor we will have to find Q  1 2 
V = − for x > 3a
potential difference across it. Or Vad . 4 πε 0  x − 3a 3a + x
Now, Va − 2i1 + 2 = Vd
V = 0 at x = 9a
or Va − Vd = 2i1 − 2 = − 2.4 V
For x < − 3a, we can write
or Vd − Va = 2.4 V = Vda
Q  1 2 
Energy stored in capacitor: V = − for x < − 3a

4 πε 0  3a − x −3a − x
1
U = CVda2
2 In this region potential will be zero only at x → − ∞
1 Thus, we can summarise it as under.
= ( 5 × 10−6 ) ( 2.4 )2
2 (i) At x = 3a , V = + ∞
= 1.44 × 10−5 J (ii) At x = − 3a , V = − ∞
40. (a) Let P ( x, y ) be a general point on x-y plane. Electric Q  1 2 
(iii) For x < − 3a,V = −
potential at point P would be, 4 πε 0  3a − x −3a − x
y
(iv) For − 3a < x < 3a, expression of V is same i.e.
V=0 Q  1 2 
V = −

4 πε 0  3a − x 3a + x
(a,0) C (5a,0) (9a,0)
O
x Q  1 2 
(v) For x > 3a , V = −
4a 
4 πε 0  x − 3a 3a + x

Potential on x-axis is zero at two places at x = a


and x = 9a .
Electrostatics 431

The V- x graph is shown below, 1 Q2


U1 = ⋅ …(i)
40πε 0 R
V
(ii) Energy stored outside the sphere (U 2 )
Electric field at a distance r is
1 Q
−3a a 9a E= ⋅
+3a
x 4 π ε 0 r2
2
1 ε  1 Q
∴ u= ε0 E 2 = 0  ⋅ 
2 2  4πε 0 r2 
dU = ( 4 πr2 dr )
(c) Potential at centre i.e. at x = 5a will be, ∴ dU = u ⋅ dV
Q 1 2 Q  ε  1 Q  2
V =  − = = positive
4 πε 0  2a 8a  16πε 0 a = ( 4 πr dr )  0 
2
⋅ 2 
 2  4 πε 0 r  
Potential on the circle will be zero.
Since, potential at centre > potential on circumference on Q 2 dr
dU =
it, the particle will cross the circle because positive 8πε 0 r2
charge moves from higher potential to lower potential. ∝ Q2 ∝ dr
Speed of particle, while crossing the circle would be, ∴ U2 = ∫ R dU = 8πε0 ⋅ ∫ R r2
2q ( ∆V ) Qq Q2
v= = U2 = …(ii)
m 8πε 0 ma 8πε 0 R
Here, ∆V is the potential difference between the centre and Therefore, total energy of the system is
circumference of the circle. Q2 Q2
U = U1 + U 2 = +
41. (a) In this case the electric field exists from centre of the 40πε 0 R 8πε 0 R
sphere to infinity. Potential energy is stored in electric
field, with energy density 3 Q2
or U=
1 20 πε 0 R
u = ε0 E 2 (Q Energy/Volume)
2 (b) Comparing this with gravitational forces, the
(i) Energy stored within the sphere (U1 ) gravitational potential energy of earth will be
Electric field at a distance r is 3 GM 2
1 Q U=−
E= ⋅ ⋅r 5 R
4 πε 0 R 3 1
by replacing Q 2 by M 2 and by G
1 ε  1 Q 
2 4 πε 0
u= ε0 E 2 = 0  ⋅ r
2 2  4 πε 0 R 3  g=
GM
R2
Volume of element,
gR 2
dV = ( 4 πr2 ) dr ∴ G=
M
Energy stored in this volume, dU = u ( dV ) −3
2 U= MgR
ε0  1 Q  5
dU = ( 4 πr2 dr )  ⋅ r Therefore, energy needed to completely disassemble the
2  4 πε 0 R 3 
earth against gravitational pull amongst its constituent
1 Q2 4 particles will be given by
dU = ⋅ ⋅ r dr
8πε 0 R 6 3
E = | U | = MgR
5
R 1 Q2 R 4
∴ U1 = ∫ 0 dU = 8πε0 R 6 ∫ 0 r dr Substituting the values, we get
3
E = (10m/s 2 ) ( 2.5 × 1031 kg -m )
Q2 5
= [ r5 ]R0
40πε 0 R 6 E = 1.5 × 1032 J
432 Electrostatics

(c) This is the case of a charged spherical conductor of ε 0 ( 2 A ) ( 8.85 × 10−12 ) ( 0.04 )
C ′A = =
1 Q2 d 8.85 × 10−4
radius R, energy of which is given by =
2C
2 2 C ′A = 0.4 × 10−9 F
1 Q Q
or U= . or U = Energy stored in this case would be
2 4 πε 0 R 8πε 0 R
1 ( q A )2 1 ( 2.2 × 10−7 )2
42. Total charge in the ring is Q = ( 2πR ) λ UA′ = =
2 C ′A 2 ( 0.4 × 10−9 )
Potential due to a ring at a distance of x from its centre on its
UA′ = 6.05 × 10−5 J > U A
1 Q
axis is given by V ( x ) = ⋅ Therefore, work done to remove the slab would be
4 πε 0 R 2 + x 2
W = U ′A − U A = ( 6.05 − 1.21) × 10−5 J
1 Q
and at the centre is Vcentre = or W = 4.84 × 10−5 J
4 πε 0 R (c) Capacity of B when filled with dielectric is
Using the above formula Kε A ( 9 ) ( 8.85 × 10−12 ) ( 0.02 )
CB = 0 =
1 2πRλ λ d ( 8.85 × 10−4 )
Vp = ⋅ =
4 πε 0 R + 3R
2 2 4 ε0 C B = 1.8 × 10−9 F
1 2 πRλ λ These two capacitors are in parallel. Therefore, net
Vo = . =
4 πε 0 R 2ε 0 capacitance of the system is
Vo > V p C = C ′A + C B = ( 0.4 + 1.8 ) × 10−9 F
Potential difference between points O and P is C = 2.2 × 10−9 F
λ λ λ Charge stored in the system is q = q A = 2.2 × 10−7 C
V = Vo − V p = − =
2ε 0 4 ε 0 4 ε 0 1 q2
Therefore, energy stored, U =
1 2 2qV 2C
∴ mv ≥ qV or v ≥ 1 ( 2.2 × 10−7 )2
2 m U= or U = 1.1 × 10−5 J
2 ( 2.2 × 10−9 )
2qλ qλ
or v≥ or v≥ 44. Let a be the side of the square
4 ε0 m 2ε 0 m
plate.
Therefore, minimum value of speed v should be As shown in figure, C1 and C 2
qλ are in parallel. Therefore, C1
v min =
2ε 0 m total capacity of capacitors in
the position shown is :
43. (a) Capacitor A is a combination of two capacitors C K and C2 x
CO in parallel. Hence, C = C1 + C 2
Kε 0 A ε 0 A ε A ε a ( a − x ) Kε 0 ax
C A = C K + CO = + = ( K + 1) 0 C= 0 +
d d d d d
Here, A = 0.02 m2 . Substituting the values, we have ε aV
∴ q = CV = 0 ( a − x + Kx )
8.85 × 10−12 ( 0.02 ) d
C A = ( 9 + 1) As plates are lowered in the oil, C increases or charge stored
( 8.85 × 10−4 )
will increase.
C A = 2.0 × 10−9 F dq ε 0 aV dx
Therefore, i = = ( K − 1) ⋅
Energy stored in capacitor A, when connected with a dt d dt
110 V battery is Substituting the values
1 1
U A = C AV 2 = ( 2 × 10−9 ) (110 )2 ε 0 = 8.85 × 10−12 C2 / N-m2
2 2
−5 a = 1m,V = 500 volt, d = 0.01 m, K = 11
U A = 1.21 × 10 J
dx
(b) Charge stored in the capacitor and = speed of plate = 0.001m/s
dt
q A = C AV = ( 2.0 × 10−9 ) (110 ) ⇒ q A = 2.2 × 10−7 C
( 8.85 × 10−12 )(1)( 500 ) (11 − 1) ( 0.001)
We get current i =
Now, this charge remains constant even after battery is ( 0.01)
disconnected. When the slab is removed, capacitance of
A will get reduced. Let it be C ′A i = 4.43 × 10–9 A
Electrostatics 433

45. Let length and breadth of the P ∴ 300 = q1 + q2 …(i)


capacitor be l and b respectively Similarly, net charge on plates 4 and 5 before joining
and d be the distance between the = net charge after joining
plates as shown in figure. Then, − 360 = − q2 − q3
consider a strip at a distance x of Q
or 360 = q2 + q3 …(ii)
width dx.
Applying Kirchhoff’s second law in closed loop ABCDA
Now, QR = x tan θ dx
θ q1 q2 q3
and PQ = d − x tan θ x
− + =0
R 3 2 2
where, tan θ = d / l
ε0 A or 2q1 − 3q2 + 3q3 = 0 …(iii)
Using the formula of C =
d − t1 − t 2 + ( t1 / K 1 ) + ( t 2 / K 2 ) Solving Eqs. (i) , (ii) and (iii), we get
We have, q1 = 90 µC , q2 = 210 µC
Small capacitance between P and R is and q3 = 150 µC
ε 0 ( bdx ) (b) (i) Electrostatic energy stored before, completing the
dC =
PQ QR circuit
d − PQ − PR + +
K1 K2 1 1
U i = ( 3 × 10−6 ) (100 )2 + ( 2 × 10−6 ) (180 )2
d 2 2
Sustituting the values of PQ and QR with tan θ =
l  1 2
QU = CV 
we have,  2 
ε 0 AK 1 K 2 = 4.74 × 10−2 J or U i = 47.4 mJ
dC = dx [ bl = A ]
d {K 2 l + ( K 1 − K 2 )x}
(ii) Electrostatic energy stored after, completing the
Now all such small capacitors from x = 0 and x = l will be in circuit
parallel as potential difference between them will be same. 1 ( 90 × 10−6 )2 1 ( 210 × 10−6 )2
Uf = +
Hence, the net capacitance of the given parallel plate 2 ( 3 × 10−6 ) 2 ( 2 × 10−6 )
capacitor is obtained by adding such infinitesimal capacitors
placed parallel from x = 0 to x = l 1 (150 × 10−6 )2  1 q2 
+ U = 
x=l l ε 0 AK 1 K 2 2 ( 2 × 10−6 )  2C
i.e. CR = ∫ dC = ∫ dx
0 d {K l + ( K − K ) x}
x=0
2 1 2 . × 10−2 J or U f = 18 mJ
= 18
K1 K 2 ε0 A K
Finally we get C R = ln 2 47. Let q1 and q2 be the charges on the two spheres before
( K 2 − K 1 )d K1
connecting them.
CK 1 K 2 K ε A
= ln 2 where, C = 0 Then, q1 = σ ( 4 πR 2 ),
K 2 − K1 K1 d
and q2 = σ ( 4 π ) ( 2R )2 = 16σπR 2
46. (a) Charge on capacitor A, before joining with an uncharged Therefore, total charge (q) on both the spheres is
capacitor
q = q1 + q2 = 20 σπR 2
q A = CV = (100 ) ( 3 )µC = 300 µC
q2 Now, after connecting, the charge is distributed in the ratio of
+ – their capacities, which in turn depends on the ratio of their
B C
3 4
radii (C = 4 πε 0 R ).

+ 2 5 q 1′ R 1
q1 q3 ∴ = =
– 1
+
6 q ′2 2R 2
q 20
∴ q1 ′ = = σπR 2
A D 3 3
Similarly, charge on capacitor B 2q 40
and q2 ′ = = σπR 2
qB = (180 ) ( 2 ) µC = 360 µC 3 3
Let q1 , q2 and q3 be the charges on the three capacitors Therefore, surface charge densities on the spheres are
after joining them as shown in figure. q′ ( 20 / 3 ) σπR 2 5
σ1 = 1 2 = = σ
(q1 , q2 and q3 are in microcoulombs) 4 πR 4 πR 2 3
From conservation of charge
q2 ′ ( 40 / 3 )σπR 2
5
Net charge on plates 2 and 3 before joining and σ2 = = = σ
4 π ( 2R )2 16πR 2 6
= net charge after joining
434 Electrostatics

Hence, surface charge density on the bigger sphere is σ 2 2Q


V = 3i1 R +
i.e. ( 5 / 6 )σ. C
2Q
48. This is basically a problem of discharging of CR circuit, or 3i1 R = V −
because between the plates of the capacitor, there is capacitor C
1  2Q 
as well as resistance. or i1 = V − 
3R  C
d  l 
R= R =  dQ 1  2Q 
σA  σA  or = V − 
dt 3R  C
Kε 0 A
and C= dQ dt
d or =
2Q 3R
∴ Time constant, C V −
C

τ c = CR = 0 Q dQ t dt
σ

R
or ∫0 2Q ∫ 0 3R
=
V −
Substituting the values, we C
have CV
This equation gives Q = (1 − e−2 t / 3 RC )
5 × 8.86 × 10−12 2
τc = = 5.98 s dQ V −2 t / 3 RC
7.4 × 10−12 (b) i1 = = e
dt 3R
Charge at any time decreases exponentially as V
q = q0 e− t / τ c V + e−2 t / 3 RC
V + i1 R 3
From Eq. (i) i = =
Here, q0 = 8.85 × 10−6 C ( Charge at time t = 0) 2R 2R
Therefore, discharging (leakage) current at time t will be ∴ Current through AB
V
given by V + e−2 t / 3 RC
V −2 t / 3 RC
 dq  q i2 = i − i1 = 3 −
i =  −  = 0 e− t / τ c 2R 3R
e
 dt  τ c
V V −2 t / 3 RC
or current at t = 12 s is i2 = − e
2R 6R
( 8.85 × 10−6 ) −12 / 5.98
i= e V
5.98 i2 = as t → ∞
2R
= 0.198 × 10−6 A = 0.198 µA
50. Capacities of conducting spheres are in the ratio of their
i = 0.198 µA
radii. Let C1 and C 2 be the capacities of S 1 and S 2 , then
49. Let at any time t charge on capacitor C be Q and currents are C2 R
=
as shown in figure. Since, charge Q will increase with time t. C1 r
Therefore, (a) Charges are distributed in the ratio of their capacities. Let
R in the first contact, charge acquired by S 2 is q1 .
A
N vvvvv S Therefore, charge on S 1 will be Q − q1 .
i1 q1 C R
+ ∴ = 2 =
Q − q1 C1
vvvvv

Q

C r
V R
R  R 
M vvvvv T ∴ q1 = Q   …(i)
i B  R + r

dQ In the second contact, S 1 again acquires the same


i1 = charge Q.
dt
Therefore, total charge in S 1 and S 2 will be
(a) Applying Kirchhoff 's second law in loop MNABM
 R 
V = ( i − i1 ) R + iR Q + q1 = Q 1 + 
 R + r
or V = 2iR − i1 R …(i)
Similarly, applying Kirchhoff ’s second law in loop This charge is again distributed in the same ratio.
MNSTM, we have Therefore, charge on S 2 in second contact,
Q  R  R   R  R  
2
V = i1 R + + iR (ii) q2 = Q  1 + =  +
C   Q   
 R + r  R + r  R + r  R + r 
Eliminating i from Eqs. (i) and (ii), we get
Electrostatics 435

 R  R   R  
2 3 ( ∆KE = 0 because K i = K f = 0 )
Similarly, q3 = Q  +  +   ∴ mgH = q [VO − V p ]
 R + r  R + r  R + r 
 q  σ 
or gH =     [a − a + H + H ]
2 2
…(i)
 R  R 
2
 R  
n
 m  2ε 0 
and qn = Q  +  + ...+   
 R + r  R + r  R + r 

q 4 ε0 g
=
m σ
R  R  
n
a (1 − rn ) qσ
or qn = Q 1 −    S n =  …(ii) ∴ = 2g
r   R + r   (1− r )  2ε 0 m
 
Therefore, electrostatic energy of S 2 after n such contacts Substituting in Eq. (i), we get
qn2 qn2 gH = 2g [ a + H − a 2 + H 2 ]
= or U n =
2 ( 4 πε 0 R ) 8πε 0 R H
or = (a + H ) − a2 + H 2
where, qn can be written from Eq. (ii). 2
R H
(b) As n → ∞ q∞ = Q or a2 + H 2 = a +
r 2
q∞2 Q 2 R 2 / r2 Q 2R H2
∴ U∞ = = or U ∞ = or a2 + H 2 = a2 + + aH
2C 8πε 0 R 8 πε 0 r2 4
3 2
51. Potential at a height H on the axis of the disc V (P) or H = aH
4
The charge dq contained in the ring shown in figure
4
P (q , m ) or H= a
3
and H=0
x H
∴ H = (4 / 3)a
(b) Potential energy of the particle at height
H = Electrostatic potential energy + gravitational
O potential energy
dr
r
∴ U = qV + mgH
a
HereV = Potential at height H
dq = ( 2πrdr ) σ σq
U= [ a 2 + H 2 − H ] + mgH …(ii)
Potential at P due to this ring 2ε 0
1 dq − dU
dV = ⋅ where x = H 2 + r2 At equilibrium position F = =0
4 πε 0 x dH
Differentiating Eq. (ii) w.r.t. H
1 ( 2πrdr )σ σ rdr
dV = ⋅ =
4 πε 0 H + r2 2 2ε 0 H 2 + r2 q  1 1 
or mg + σ   ( 2H ) − 1 = 0
2 ε 0  2 a +H
2 2

∴ Potential due to the complete disc 
r =a
 σq 
Vp = ∫ r = 0 dV Q = 2 mg 
 2ε 0 
σ r =a rdr
=
2ε 0 ∫r = 0 H 2 + r2
 H 
∴ mg + 2mg  − 1 = 0
σ  a 2 + H 2 
Vp = [ a2 + H 2 − H ]
2ε 0 or 1+
2H
− 2= 0
Potential at centre, (O) will be a + H2
2

σa 2H H2 1
VO = (H = 0 ) or = 1 or =
2ε 0 a + H2
2
a +H
2 2 4
(a) Particle is released from P and it just reaches point O. a
Therefore, from conservation of mechanical energy or 3H 2 = a 2 or H =
3
Decrease in gravitational potential energy = Increase in
electrostatic potential energy From Eq. (ii), we can write
436 Electrostatics

U − H equation as ∴ Electric field at point P is


  −1  27 −3 / 2
U = mg ( 2 a + H − H ) (Parabolic variation)
2 2
dV 
E=− = 1.8 × 104 ( 8 )    + x2 
U = 2mga at H = 0 dX   2   2 

U −3 / 2 
 1  3 2 
−   + x   ( 2x )
 2  2  
2mga
E = 0 on x-axis where x = 0 and where,
√3mga 8 1
or 3/ 2
= 3/ 2
 27 2  3 2
 +x   +x 
H  2  2 
O H =a
√3 ( 4 )3 / 2 1
a ⇒ =
and U = U min = 3mga at H =  27 2
3/ 2
 3 2
3/ 2
3  +x   +x 
 2  2 
Therefore, U − H graph will be as shown
 27 2 3 2
a ⇒  +x  =4 +x 
Note that at H = , U is minimum.  2  2 
3
5
a This equation gives, x = ± m
Therefore, H = is stable equilibrium position. 2
3
The least value of kinetic energy of the particle at infinity
52. 5
y should be enough to take the particle upto x = + m
2
+√27/2 µ B + Q 5
because at x = + m, E = 0.
2
+√3/2 µ A – q
⇒ Electrostatic force on charge q is zero or Fe = 0.
V0 m
O x 5
x P q0 For at x > m, E is repulsive (towards positive x-axis)
2
–√3/2 µ C – q
5
and for x < m, E is attractive (towards negative x-axis)
–√27/2 µ D +Q 2
5
Now, from Eq. (i), potential at x = m
In the figure, q = 1 µC = 10−6 C, q0 = + 01
. µC = 10−7 C 2
 
and m = 6 × 10−4 kg and Q = 8 µC = 8 × 10−6 C  8 1 
Let P be any point at a distance x from origin O. Then V = 1.8 × 10 
4
− 
3  27 + 5 3 5
+
AP = CP = + x2  2 2 2 2 
2
V = 2.7 × 10 Volt
4
27
BP = DP = + x2 5
2 Applying energy conservation at x = ∞ and x = m
2
2KQ 2Kq
Electric potential at point P will be, V = − 1 2
BP AP mv0 = q0V …(ii)
2
1
where, K = = 9 × 109 Nm2 / C2 2q0V
4 πε 0 ∴ v0 =
m
  Substituting these values,
 8 × 10−6 10 −6 
∴ V = 2 × 9 × 109  −  2 × 10−7 × 2.7 × 104
 27 + x2 v0 = ⇒ v0 = 3 m/s
+ x2 
3
6 × 10− 4
 2 2 
∴ Minimum value of v0 is 3 m/s
 
  From Eq. (i), potential at origin ( x = 0 ) is
8 1
V = 1.8 × 104  −  …(i)  8 − 1
 27 + x2 3
+ x2  V0 = 18. × 104  27 3  ≈ 2.4 × 10 V
4

 2 2   
 2 2
Electrostatics 437

Let K be the kinetic energy of the particle at origin. Therefore, minimum horizontal velocity imparted to the
Applying energy conservation at x =`0 and at x = ∞ lower ball, so that it can make complete revolution, is
1 2 5.86 m/s.
K + q0V0 = mv0
2 54. (a) Applying energy conservation principle,
1 2 increase in kinetic energy of the dipole = decrease in
But mv0 = q0V [from Eq. (ii)] electrostatic potential energy of the dipole.
2
K = q0 (V − V0 ) ∴ Kinetic energy of dipole at distance d from origin
K = (10−7 ) ( 2.7 × 104 − 2.4 × 104 ) = Ui − U f
K = 3 × 10− 4 J or KE = 0 − ( − p⋅ E ) = p⋅ E
5  1 q $ qp
NOTE E = 0 or Fe on q 0 is zero at x = 0 and x = ± m. Of = ( pi$ ) ⋅  i =
2  4 πε 0 d 2  4 πε 0 d 2
5
these x = 0 is stable equilibrium position and x = ± is
2 (b) Electric field at origin due to the dipole,
unstable equilibrium position. 1 2p $
−6 E= i ( Eaxis ↑ ↑ p)
53. Given : q = 1µC = 10 C Fe 4 πε 0 d 3
m = 2 × 10−3 kg v T=0 ∴ Force on charge q
and l = 0.8m pq $i
mg F = qE =
Let u be the speed of the particle at 2πε 0 d 3
its lowest point and v its speed at
q 55. Electric field near a large metallic plate is given by
highest point.
E = σ / ε 0 . In between the plates the two fields will
At highest point three forces are be in opposite directions. Hence,
acting on the particle.
σ − σ2
(a) Electrostatic repulsion
u E net = 1 = E0 (say)
ε0
1 q2
Fe = (outwards) Now, W = ( q ) (potential difference)
4 πε 0 l 2
= q ( E0 a cos 45° )
(b) Weight w = mg (inwards)
σ − σ2   a 
(c) Tension T (inwards) = (q )  1   
 ε0   2 
T = 0, if the particle has just to complete the circle and the
necessary centripetal force is provided by w − Fe i.e. (σ1 − σ 2 ) qa
=
mv2
l  1 q 2 2ε 0
= w − Fe or v2 =  mg − 
l m  4 π ε0 l2  56. Let q be the charge on the bubble, then
 9.0 × 10 × (10 ) 9 −6 2  Kq  1 
v2 =
0.8 −3
 2 × 10 × 10 −  m /s
2 2 V =  Here, K = 
2 × 10 3
( 0.8 )2  a  4 πε 0 
Va
or v2 = 2.4 m2 /s 2 …(i) ∴ q=
K
Now, the electrostatic potential energy at the lowest and Let after collapsing, the radius of droplet becomes R, then
highest points are equal. Hence, from conservation of equating the volume, we have
mechanical energy. 4
Increase in gravitational potential energy ( 4 πa 2 ) t = π R 3
3
= Decrease in kinetic energy. ∴ R = ( 3a 2 t )1 / 3
1
or mg ( 2l ) = m ( u 2 − v2 ) Kq
2 Now, potential of droplet will be V ′ =
R
or u 2 = v2 + 4 gl Substituting the values, we have
2
Substituting the values of v from Eq. (i), we get Va 
(K )   1/ 3
u 2 = 2.4 + 4 (10 ) ( 0.8 ) = 34.4 m2 /s 2 K  a
V′ = or V ′ = V  
2 1/ 3
( 3a t )  3t 
∴ u = 5.86 m/s
438 Electrostatics

λ t
57. R1 dλ σ
vvvvvv ∴ ∫ λ = − ε ∫ dt
λi 0

Solving this equation, we get

vvvvvv
V C R2 σ
− t
λ = λ ie ε

σ
 λ  σλ i − ε t
Q0 is the steady state charge stored in the capacitor. Now, j = σE = σ  = e
 2πεr 2πεr
Q0 = C [potential difference across capacitor in steady state] σ
− t
= C [steady state current through R2 ] ( R2 ) j = ji e ε

 V 
=C   ⋅ R2 Thus j decreases exponentially with time.
 R1 + R2 
2. Using Gauss’ law at outer surface, let charge on dipole is q ,
C V R2
∴ Q0 = Σq 1
R1 + R2 φ= = E ⋅ A or E = Σq
ε0 Aε 0
1
α is 1/ τ c or ( +Q + q − q ) Q σ
C R net = = = = constant
Aε 0 Aε 0 ε 0
R1 3. Polarity should be mentioned in the question. Potential on
vvvvvv
each of them can be zero if, qnet = 0
or q1 ± q2 = 0
vvvvvv

R2 or 120C1 ± 200C 2 = 0
or 3C1 ± 5C 2 = 0
4. Balls will gain positive charge and hence move towards
negative plate.
Here, R net is equivalent resistance across capacitor after
On reaching negative plate, balls will attain negative charge
short circuiting the battery. Thus,
and come back to positive plate.
R1 R2
R net = (As R1 and R2 are in parallel) So, balls will keep on oscillating.
R1 + R2 But oscillation is not SHM,
α=
1 as force on balls is not ∝ x.
 RR  ∴ (d) is correct.
C 1 2 
 R1 + R2  5. As the balls keep on carrying charge from one plate to
R + R2 another, current will keep on flowing. At bottom plate
= 1
C R1 R2 kq  1 
= V0 ∴ k = 
r  4 πε 0 
Section-3 ⇒ q=
V0 r
1. Here charge per unit length λ is not constant and there is k
some conductivity of the material between line charge and Inside the cylinder, electric field E = [V0 − ( − V0 )] h
shell. So free electrons will start flowing opposite to the = 2V0 h.
electric field, which is radially outwards and value of this ⇒ Acceleration of each ball,
electric field at a distance r is,
qE 2hr 2
λ a= = ⋅V0
E= ( λ ≠ constant ) m km
2πεr
λ ⇒ Time taken by the balls to reach the other plate,
≠ 2h 2h. k m 1 km
2πε 0 r t= = =
Since free electrons are flowing radially inwards, so
a 2hrV02 V0 r
positive line charge will gradually decrease. Now, If there are n balls, then
dq nq Vr r
I=− = j ( A ) = (σE ) A [as j = σE ] Average current, iav = = n × 0 × V0
dt t k km
d λ
∴ − ( λl ) = (σ )( 2πrl ) [ q = λl ] ⇒ iav ∝ V02
dt 2πεr

You might also like